OBM universitária

2001-08-09 Por tôpico Bruno Leite

Gostaria de ter alguma noção sobre o que será pedido na OBM universitária.
Por acaso as questões vão se limitar a Análise, Álgebra e Combinatória? Ou
Geometria também?
A prova de 1º de setembro vai ter só testes, ou questões escritas?

Bruno Leite
http://www.ime.usp.br/~brleite





Re: gauss

2001-08-04 Por tôpico Bruno Leite

Tem a demonstração elementar de Erdos e Selberg, que eu não sei onde você
pode achar (em algum jornal especializado de 1949/1950, quem sabe?); e tem a
demonstração analítica, que pode ser encontrada em vários livros de teoria
analítica dos números. Por exemplo, no livro

T. M. Apostol
Introduction to Analytic Number Theory

Talvez tenha no Hardy e Wright, mas eu não lembro.

Bruno Leite
www.ime.usp.br/~brleite


-Mensagem original-
De: pirotron <[EMAIL PROTECTED]>
Para: [EMAIL PROTECTED] <[EMAIL PROTECTED]>
Data: Sábado, 4 de Agosto de 2001 19:46
Assunto: gauss


>SOU MUI GRATO A TODOS,O PESSOAL ESTA DE PARABÉNS.ALGUÉM
>POR ACASO SABE ONDE ENCONTRAR A DEMONSTRAÇÃO DA
>DISTRIBUIÇÃO DOS N PRIMOS PI(X)~X/LOGX.
>
>
>
>ATENCIOSAMENTE, O INTEGRALMENTE GRATO.
>
>
>__
>Acesso pelo menor preço do mercado! R$ 14,90 nos 3 primeiros meses!
>ASSINE AGORA! http://www.bol.com.br/acessobol/
>
>




Re: fermat teorema

2001-08-01 Por tôpico Bruno Leite

A solução de Wiles foi publicada no Annals of Mathematics. É um artigo de
umas 70 páginas, de 1994 ou 1995.

Nunca vi uma versão em português.

Bruno


-Mensagem original-
De: pirotron <[EMAIL PROTECTED]>
Para: [EMAIL PROTECTED] <[EMAIL PROTECTED]>
Data: Quarta-feira, 1 de Agosto de 2001 18:41
Assunto: fermat teorema


>quero agradecer pela ajuda .alguém tem a solução q wiles
>apresentou para o último teorema de fermat?ou pelo menos
>onde eu acho ,em portugues de preferencia.
>
>
>SAUDAÇÕES QUâNTICAS
>
>
>__
>Acesso pelo menor preço do mercado! R$ 14,90 nos 3 primeiros meses!
>ASSINE AGORA! http://www.bol.com.br/acessobol/
>
>




Re: SELEÇÃO IMO

2001-08-01 Por tôpico Bruno Leite

Linda solução! Nunca tinha conseguido fazer essa questão...

Bruno

-Mensagem original-
De: Marcelo Rufino de Oliveira <[EMAIL PROTECTED]>
Para: [EMAIL PROTECTED] <[EMAIL PROTECTED]>
Data: Quarta-feira, 1 de Agosto de 2001 03:25
Assunto: Re: SELEÇÃO IMO


>Na segunda questão faça o seguinte:
>
>2) se a = sqrt(4-sqrt5-a), b = sqrt(4+sqrt5-b), c = sqrt(4-sqrt5+c) e
>d = sqrt(4+sqrt5+d), calcule a*b*c*d.
>
>Solução:
>
>Inicialmente note que, devido as equações que definem a, b, c e d, então
>estes valores são todos distintos.
>
>Elevando ao quadrado duas vezes as equações obtemos:
>
>(1) (a^2 - 4)^2 = 5 - a   =>   a^4 - 8a^2 + 16 = 5 - a   =>
>a^4 - 8a^2 + a + 11 = 0
>
>(2) (b^2 - 4)^2 = 5 - b   =>   b^4 - 8b^2 + 16 = 5 - b   =>
>b^4 - 8b^2 + b + 11 = 0
>
>(3) (c^2 - 4)^2 = 5 + c   =>   c^4 - 8c^2 + 16 = 5 + c   =>
>c^4 - 8c^2 - c + 11 = 0
>
>(4) (d^2 - 4)^2 = 5 + d   =>   d^4 - 8d^2 + 16 = 5 + d   =>
>d^4 - 8d^2 - d + 11 = 0
>
>Assim, a e b são 2 das 4 raízes do polinômio  P(x) = x^4 - 8x^2 + x + 11 e
>c e d são 2 das 4 raízes do polinômio Q(x) = x^4 - 8x^2 - x + 11.
>
>Aplicando x = - c em P(x) temos:
>P(- c) = c^4 - 8c^2 - c + 11 = 0   =>   - c é raiz de P(x).
>
>Aplicando x = - d em P(x) temos:
>P(- d) = d^4 - 8d^2 - d + 11 = 0   =>   - d é raiz de P(x).
>
>Deste modo, as raízes de P(x) são  a, b, - c e - d.
>
>Como a multiplicação das raízes P(x) é igual a  11,
>temos que  a.b.(- c)(- d) = 11   =>   abcd = 11.
>
>
>Falou,
>Marcelo Rufino de Oliveira
>
>
>- Original Message -
>From: Henrique Lima <[EMAIL PROTECTED]>
>To: <[EMAIL PROTECTED]>
>Sent: Tuesday, July 31, 2001 11:16 PM
>Subject: SELEÇÃO IMO
>
>
>>
>>   alguém pode ajudar nesses problemas?
>> 1)se m e n são inteiros positivos tais q 2^n  - 1 divide m^2 +9, prove q
n
>> eh uma potencia de 2
>> se n eh uma potencia de 2 prove q existe um inteiro m (positivo) tal q
2^n
>> -1 divide m^2 + 9
>> 2)se a=sqrt(4-sqrt5-a), b=sqrt(4+sqrt5-b), c=sqrt(4-sqrt5+c) e
>> d=sqrt(4+sqrt5+d), calcule a*b*c*d
>> 3)sejam Q+ e Z os conjuntos dos racionais estritamente positivos e o
>> conjunto dos inteiros. determine todas as funções f:Q+ ->Z satisfazendo
as
>> seguintes condições:
>> (i)f(1999)=1
>> (ii)f(ab)=f(a)+f(b) ,pra qq a,b racionais estritamente positivos
>> (iii)f(a+b)>=min{f(a),f(b)}, pra qq a,b racionais estritamente positivos
>>
>> valeu!
>>
>> _
>> Get your FREE download of MSN Explorer at
http://explorer.msn.com/intl.asp
>>
>>
>




Re: Problema Republica Tcheca

2001-07-20 Por tôpico Bruno Leite

Ui, agora que estou vendo que me esqueci de explicar de onde veio esta
fórmula z(n)=n+[n/4]+[n/8]+[n/16]+...
Obrigado por ter explicado!

Mas foi assim mesmo que eu pensei, usando a mesma idéia que Lagrange(foi ele
mesmo?) usou para calcular qual a maior potência de um primo que divide n!

Sobre o chute para 82308, eu fiz bem parecido com você, primeiro calculei
para 82304 ou 82305 e rapidamente cheguei ao meu "chute".

Bruno

-Mensagem original-
De: Ralph Costa Teixeira <[EMAIL PROTECTED]>
Para: [EMAIL PROTECTED] <[EMAIL PROTECTED]>
Data: Sexta-feira, 20 de Julho de 2001 17:26
Assunto: Re: Problema Republica Tcheca


>>> Seja z(n)=n+[n/4]+[n/8]+[n/16]+...
>
>Legal sua solução, Bruno. Voce definiu g(n) como "o numero de vezes que
>2 divide n". Então o somatório de g(k) de 1 até n é o número de "2" que
>dividem todos os números de 1 até k, por assim dizer. Você pode calcular
>isso assim: olhe para os números de 1 a k e
>
>conte 1 para cada número par
>conte 1 extra para os múltiplos de 4
>conte 1 mais para os múltiplos de 8
>...
>
>Chegando à sua fórmula:
>
>SUM(g(k), 1 a n) = [n/2]+[n/4]+[n/8]+...
>
>Para f(n), adicione uma linha às regras:
>
>conte 1 para cada ímpar
>
>EnTão SUM(f(k), 1 a n) = [(n+1)/2]+[n/2]+[n/4]+[n/8]+[n/16]+...
>
>que corresponde à sua fórmula para z(n) quando n é par.
>
>>> Fazendo umas continhas, vemos que z(82308)=123457
>
>E aqui, não sei qual método você usou para achar o 82308, mas eu
pensava
>em algo assim para ajudar:
>
>z(n) <= n + n/4 + n/8 + n/16 + ... = n + n/2 = 3n/2
>
>Então n >= 2z(n)/3.
>
>Se z(n)=123457, então n>=2.123457/3 = 82304,???. Assim, n=82305 é um
>ótimo lugar para começar a busca...
>
>Abraço,
>Ralph
>
>
>




Re: O próximo número é ...

2001-07-19 Por tôpico Bruno Leite

Na minha opinião, qualquer número que você colocar serve. Esse tipo de
questão não é muito preciso.

Mesmo se fosse 2,4,6,8,10,...qual o próximo termo? Poderia ser 12 ou
raiz(77)  ou qualquer coisa!

Tenho certeza que, dados uns termos iniciais de uma sequência, pode-se
arrumar umas 3 ou 4  "regras" distintas que nos forneçam 3 ou quatro
respotas "com alguma lógica".

Por exemplo, poderíamos completar 2,4,6,8,10 como

2,4,6,8,10,10,8,6,4,2,2,4,6,8,10,10,8,6,4,2...
ou
2,4,6,8,10,20,40,60,80,100,200,400,600,800,1000...
ou
2,4,6,8,10,4,6,8,10,12,6,8,10,12,14...
ou sei lá...

Ainda poderíamos fazer interpolação polinomial com os dados!

Todas essas têm um certo padrão.

Bruno Leite
[EMAIL PROTECTED]
[EMAIL PROTECTED]
http://www.ime.usp.br/~brleite


-Mensagem original-
De: [EMAIL PROTECTED] <[EMAIL PROTECTED]>
Para: [EMAIL PROTECTED] <[EMAIL PROTECTED]>
Data: Quinta-feira, 19 de Julho de 2001 15:50
Assunto: O próximo número é ...


   Olá a todos,
   Será que alguém pode me ajudar com essa ?
Tem-se a seguinte seqüência: 80, 84, 72, 27. Sabe-se que os termos dessa
seqüência não foram postos ao acaso e, portanto, há uma regra que indica o
próximo termo da progressão. Sabendo-se que esta regra se baseia em
conceitos
aritméticos, diga qual o próximo termo dessa seqüência.
a-) 97
b-) 81
c-) -81
d-) -97
e-) 91
   Obrigado pela atenção. Abraços,
 Raul










Re: Problema Republica Tcheca

2001-07-18 Por tôpico Bruno Leite


-Mensagem original-
De: Bruno Leite <[EMAIL PROTECTED]>
Para: [EMAIL PROTECTED] <[EMAIL PROTECTED]>
Data: Quinta-feira, 19 de Julho de 2001 01:47
Assunto: Re: Problema Republica Tcheca


>
>-Mensagem original-
>De: Bruno Leite <[EMAIL PROTECTED]>
>Para: [EMAIL PROTECTED] <[EMAIL PROTECTED]>
>Data: Quinta-feira, 19 de Julho de 2001 00:46
>Assunto: Re: Problema Republica Tcheca
>
>
>>Oi Henrique
>>
>>Vamos definir a função g:N->N, mais natural e mais fácil de se lidar, da
>>seguinte forma:
>>g(n)=k pra todo inteiro com n =2^k*l , onde k eh um numero natural e l eh
>>impar. veja que ela é quase igual a função f, só diferindo nos ímpares
>>(f(impar)=1 e g(impar)=0).
>>
>>Agora vamos calcular os valores iniciais de g e dispo-los de uma forma
>>diferente.
>>
>>Considere a árvore abaixo:
>>
>>g(2)
>>g(3) g(4)
>>g(5) g(6)   g(7)g(8)
>>g(9) g(10) g(11) g(12) g(13) g(14) g(15) g(16)
>>
>>Cada linha tem 2^(n-1) elementos. Vamos dizer que g(2) é pai de g(3) e
>g(4),
>>assim como g(5) é pai de g(9) e g(10) e g(8) é pai de g(15) e g(16). de
>modo
>>geral, g(n) é pai de g(2n-1) e g(2n).
>>
>>Substituindo pelos valores de g:
>>
>>1
>>0   2
>>01 03
>>0 1 0 2 0 1 0 4
>>
>>É fácil ver e provar que se g(n)=k, então os filhos de g(n) valem 0 e k+1.
>A
>>partir daí vc sabe dizer quanto dá a soma de cada linha?
>>
>>Não vou terminar o problema por preguiça e porque tenho certeza que agora
>>não falta muito, até porque é fácil relacionar as somas de f com as de
g...
>
>Não sei se eu fui claro, então vou dar uma consertada.
>
>A soma da i-esima linha da arvore da função g é 2^(i-1) (indução)
>
>Então a soma das linas da arvore até a linha k é 1+2+4+...+2^(k-1)=2^k -1
>
>Se fizéssemos a arvore de f em vez de g, todos os zeros virariam uns. Então
>a soma até a linha k é
>2^k + 2^(k-1) - 1, pois há 2^(k-1) zeros até a linha k ( vamos lembrar que
>f(1) não está na árvore, mas é um zero que se converte em um, e deve ser
>contado)
>
>
>Isso mostra que f(1)+f(2)+...+f(2^k)=2^k + 2^(k-1) - 1
>Em particular, f(1)+f(2)+...+f(2^16)=2^16 + 2^15 - 1=98303
>
>Logo sabemos que o n do problema está entre 2^16 e 2^17. Agora só falta
>somar o começo da linha 17 até dar 123456.
>
>Agora não é difícil terminar.
>

>Estou vendo **agora** que tem outro modo de se fazer este problema: fixe n.
>Quantos k, de 1  a n, satisfazem
>f(k)=1? e f(k)=2? Seja h_i o número de inteiros de 1 a n com f(k)=i. Então
>
>f(1)+...+f(n)=h_1 + 2h_2 + 3h_3 +... ( a soma é infinita, mas a partir de
um
>ponto os termos sao todos nulos)
>
>Com essa formula em mente podemos achar por tentativa  e erro o valor de n
>pedido. Vou fazer as contas com mais cuidado no papel, mas acredito que
isso
>está certinho.

Fui pensar e descobri que esse método acima não é bom. O primero jeito que
eu sugeri está correto.

Há um terceiro modo. Seja g a função definida lá no começo do email. Sabemos
que, para n par,
soma(g(k),k=1...n)=[n/2]+[n/4]+[n/8]+[n/16]+...=n/2+[n/4]+[n/8]+[n/16]+...

Então,
soma(f(k),k=1...n)=n/2+n/2+[n/4]+[n/8]+[n/16]+...=n+[n/4]+[n/8]+[n/16]+...

Agora sim, com esta fórmula certamente achamos o n pedido.

Seja z(n)=n+[n/4]+[n/8]+[n/16]+...
Fazendo umas continhas, vemos que z(82308)=123457

Logo
soma(f(k),k=1...82308)=123457=soma(f(k),k=1...82307)+f(82308)=soma(f(k),k=1.
..82307)+f(82308)=

soma(f(k),k=1...82307)+2

Logo soma(f(k),k=1...82307)=123455 e n=82307

Espero que não esteja errado pois mandar um quarto email para consertar as
coisas já é muita vergonha...

Bruno


>Bruno Leite
>[EMAIL PROTECTED]
>[EMAIL PROTECTED]
>http://www.ime.usp.br/~brleite
>
>
>
>>Bruno Leite
>>[EMAIL PROTECTED]
>>[EMAIL PROTECTED]
>>http://www.ime.usp.br/~brleite
>>
>>PS Aliás toda a lista de "Olimpíadas ao redor do mundo" é interessante.
>>Gostaria de saber se alguém sabe fazer o 72.
>>
>>-Mensagem original-
>>De: Henrique Lima <[EMAIL PROTECTED]>
>>Para: [EMAIL PROTECTED] <[EMAIL PROTECTED]>
>>Data: Quinta-feira, 19 de Julho de 2001 00:07
>>Assunto: Problema Republica Tcheca
>>
>>
>>> gostaria de ajuda nesse problema
>>>
>>>Uma função f:N->N é tal q f(n)=1 se n eh ímpar e f(n)=k pra todo inteiro
>>par
>>>n =2^k*l , onde k eh um numero natural e l eh impar. determine o maior
>>>natural n para o qual:
>>>  f(1)+f(2)+...+f(n)=<123456
>>>
>>>valeuz
>>>
>>>_
>>>Get Your Private, Free E-mail from MSN Hotmail at http://www.hotmail.com.
>>>
>>
>




Re: Problema Republica Tcheca

2001-07-18 Por tôpico Bruno Leite


-Mensagem original-
De: Bruno Leite <[EMAIL PROTECTED]>
Para: [EMAIL PROTECTED] <[EMAIL PROTECTED]>
Data: Quinta-feira, 19 de Julho de 2001 00:46
Assunto: Re: Problema Republica Tcheca


>Oi Henrique
>
>Vamos definir a função g:N->N, mais natural e mais fácil de se lidar, da
>seguinte forma:
>g(n)=k pra todo inteiro com n =2^k*l , onde k eh um numero natural e l eh
>impar. veja que ela é quase igual a função f, só diferindo nos ímpares
>(f(impar)=1 e g(impar)=0).
>
>Agora vamos calcular os valores iniciais de g e dispo-los de uma forma
>diferente.
>
>Considere a árvore abaixo:
>
>g(2)
>g(3) g(4)
>g(5) g(6)   g(7)g(8)
>g(9) g(10) g(11) g(12) g(13) g(14) g(15) g(16)
>
>Cada linha tem 2^(n-1) elementos. Vamos dizer que g(2) é pai de g(3) e
g(4),
>assim como g(5) é pai de g(9) e g(10) e g(8) é pai de g(15) e g(16). de
modo
>geral, g(n) é pai de g(2n-1) e g(2n).
>
>Substituindo pelos valores de g:
>
>1
>0   2
>01 03
>0 1 0 2 0 1 0 4
>
>É fácil ver e provar que se g(n)=k, então os filhos de g(n) valem 0 e k+1.
A
>partir daí vc sabe dizer quanto dá a soma de cada linha?
>
>Não vou terminar o problema por preguiça e porque tenho certeza que agora
>não falta muito, até porque é fácil relacionar as somas de f com as de g...

Não sei se eu fui claro, então vou dar uma consertada.

A soma da i-esima linha da arvore da função g é 2^(i-1) (indução)

Então a soma das linas da arvore até a linha k é 1+2+4+...+2^(k-1)=2^k -1

Se fizéssemos a arvore de f em vez de g, todos os zeros virariam uns. Então
a soma até a linha k é
2^k + 2^(k-1) - 1, pois há 2^(k-1) zeros até a linha k ( vamos lembrar que
f(1) não está na árvore, mas é um zero que se converte em um, e deve ser
contado)


Isso mostra que f(1)+f(2)+...+f(2^k)=2^k + 2^(k-1) - 1
Em particular, f(1)+f(2)+...+f(2^16)=2^16 + 2^15 - 1=98303

Logo sabemos que o n do problema está entre 2^16 e 2^17. Agora só falta
somar o começo da linha 17 até dar 123456.

Agora não é difícil terminar.

Estou vendo **agora** que tem outro modo de se fazer este problema: fixe n.
Quantos k, de 1  a n, satisfazem
f(k)=1? e f(k)=2? Seja h_i o número de inteiros de 1 a n com f(k)=i. Então

f(1)+...+f(n)=h_1 + 2h_2 + 3h_3 +... ( a soma é infinita, mas a partir de um
ponto os termos sao todos nulos)

Com essa formula em mente podemos achar por tentativa  e erro o valor de n
pedido. Vou fazer as contas com mais cuidado no papel, mas acredito que isso
está certinho.

Bruno Leite
[EMAIL PROTECTED]
[EMAIL PROTECTED]
http://www.ime.usp.br/~brleite



>Bruno Leite
>[EMAIL PROTECTED]
>[EMAIL PROTECTED]
>http://www.ime.usp.br/~brleite
>
>PS Aliás toda a lista de "Olimpíadas ao redor do mundo" é interessante.
>Gostaria de saber se alguém sabe fazer o 72.
>
>-Mensagem original-
>De: Henrique Lima <[EMAIL PROTECTED]>
>Para: [EMAIL PROTECTED] <[EMAIL PROTECTED]>
>Data: Quinta-feira, 19 de Julho de 2001 00:07
>Assunto: Problema Republica Tcheca
>
>
>> gostaria de ajuda nesse problema
>>
>>Uma função f:N->N é tal q f(n)=1 se n eh ímpar e f(n)=k pra todo inteiro
>par
>>n =2^k*l , onde k eh um numero natural e l eh impar. determine o maior
>>natural n para o qual:
>>  f(1)+f(2)+...+f(n)=<123456
>>
>>valeuz
>>
>>_
>>Get Your Private, Free E-mail from MSN Hotmail at http://www.hotmail.com.
>>
>




Re: Problema Republica Tcheca

2001-07-18 Por tôpico Bruno Leite

Oi Henrique

Vamos definir a função g:N->N, mais natural e mais fácil de se lidar, da
seguinte forma:
g(n)=k pra todo inteiro com n =2^k*l , onde k eh um numero natural e l eh
impar. veja que ela é quase igual a função f, só diferindo nos ímpares
(f(impar)=1 e g(impar)=0).

Agora vamos calcular os valores iniciais de g e dispo-los de uma forma
diferente.

Considere a árvore abaixo:

g(2)
g(3) g(4)
g(5) g(6)   g(7)g(8)
g(9) g(10) g(11) g(12) g(13) g(14) g(15) g(16)

Cada linha tem 2^(n-1) elementos. Vamos dizer que g(2) é pai de g(3) e g(4),
assim como g(5) é pai de g(9) e g(10) e g(8) é pai de g(15) e g(16). de modo
geral, g(n) é pai de g(2n-1) e g(2n).

Substituindo pelos valores de g:

1
0   2
01 03
0 1 0 2 0 1 0 4

É fácil ver e provar que se g(n)=k, então os filhos de g(n) valem 0 e k+1. A
partir daí vc sabe dizer quanto dá a soma de cada linha?

Não vou terminar o problema por preguiça e porque tenho certeza que agora
não falta muito, até porque é fácil relacionar as somas de f com as de g...

Bruno Leite
[EMAIL PROTECTED]
[EMAIL PROTECTED]
http://www.ime.usp.br/~brleite

PS Aliás toda a lista de "Olimpíadas ao redor do mundo" é interessante.
Gostaria de saber se alguém sabe fazer o 72.

-Mensagem original-
De: Henrique Lima <[EMAIL PROTECTED]>
Para: [EMAIL PROTECTED] <[EMAIL PROTECTED]>
Data: Quinta-feira, 19 de Julho de 2001 00:07
Assunto: Problema Republica Tcheca


> gostaria de ajuda nesse problema
>
>Uma função f:N->N é tal q f(n)=1 se n eh ímpar e f(n)=k pra todo inteiro
par
>n =2^k*l , onde k eh um numero natural e l eh impar. determine o maior
>natural n para o qual:
>  f(1)+f(2)+...+f(n)=<123456
>
>valeuz
>
>_
>Get Your Private, Free E-mail from MSN Hotmail at http://www.hotmail.com.
>




Re: olá e problemas :)

2001-07-18 Por tôpico Bruno Leite


-Mensagem original-
De: Fernanda Medeiros <[EMAIL PROTECTED]>
Para: [EMAIL PROTECTED] <[EMAIL PROTECTED]>
Data: Quarta-feira, 18 de Julho de 2001 23:54
Assunto: olá e problemas :)


>   Olá pessoal,
>Tudo bem? Bem, essa é a primeira mensagem que envio e peço já
>antecipadamente desculpas pelas dúvidas triviais que apresentarei agora,
>tenho dúvida em algumas dessas questões, caso alguém possa ajudar, ficarei
>imensamente grata!
>  1.Seja f(x)=x^3 -3x +1 .Determine o n° de soluções reais distintas da
>equação f(f(x))=0
>  2.Determine todos os nºs reais x tais que
>x[x[x[x]]]=88  onde [x] é o maior inteiro que não supera x
>  3.As bissetrizes dos angulos A e B do triangulo ABC intersectam os lados
>BC e AC nos pontos D e E respectivamente. Supondo que AE+BD=AB,determine a
>medida do angulo C.
>  4.Determine todas as funções estritamente crescentes f:N*->N* tais que
>f(n+f(n))=2f(n)
>  5.Mostre que todo nº racional positivo pode ser representado sob a forma
>r=a^3 + b^3/c^3 + d^3   a,b,c,d inteiros positivos

Acho que faltou (): 5.Mostre que todo nº racional positivo pode ser
representado sob a forma
r=(a^3 + b^3)/(c^3 + d^3)   a,b,c,d inteiros positivos

Estou pensando bastante nesse problema, estou muito interessado na solução.
Se alguém achar...

Bruno Leite
>  6.Determine todas as funções f:R->R que possuem a propriedade:
>   f(xf(x)+f(y))=(f(x))^2 +y
>  pra todos os reais x e y.
>
>7. O que diz o teorema das bissetrizes?
>
>  Espero que vocês me ajudem pois realmente preciso! Obrigada
>B-jinhos
> Fê
>_
>Get Your Private, Free E-mail from MSN Hotmail at http://www.hotmail.com.
>




Re: RES: [imo-problems] IMO 2001 Problems, First Day

2001-07-09 Por tôpico Bruno Leite


-Mensagem original-
De: Paulo Santa Rita <[EMAIL PROTECTED]>
Para: [EMAIL PROTECTED] <[EMAIL PROTECTED]>
Data: Segunda-feira, 9 de Julho de 2001 17:08
Assunto: Re: RES: [imo-problems] IMO 2001 Problems, First Day


>Ola Marcio,
>Ola Pessoal,
>
>SALVO MELHOR JUIZO, nao acho que o caminho que voce delineou abaixo seja
bom
>... Eu agora estou sem tempo para pensar com mais acuidade, mas, em minha
>opiniao, voce deve partir da conhecida desigualdade entre as medias
>aritmetica e geometrica :
>
>(a + b)/2 >= raiz_2(a*b)
>
>Daqui : a + b >= 2*raiz_2(a*b) => a + b + c >= c + 2*raiz_2(a*b)
>a + b + c >= c + raiz_2(2)*raiz_2(a*b).
>a + b + c >= raiz_2(c^2 + 2*a*b)
>1/(a + b + c) <= 1/raiz_2(c^2 + 2*a*b)
>
>c/(a + b + c) <= c/raiz_2(c^2 + 2*a*b)... Desigualdade (1)
>
>Agora, partindo de :
>(b + c)/2 >= raiz_2(b*c) e repetindo o raciocinio, voce chegara a
>a/(a + b + c) <= a/raiz_2(a^2 + 2*b*c) ... Desigualdade (2)
>
>E, finalmente, partindo de :
>(a + c)/2 >= raiz_2(a*c) e repetindo o raciocinio, voce chegara a
>b/(a + b + c) <= b/raiz_2(b^2 + 2*a*c) ... Desigualdade (3)
>
>Somando, membro a membro, as tres desigualdades :
>
>1<=c/raiz_2(c^2 + 2*a*b)+ a/raiz_2(a^2 + 2*b*c)+ b/raiz_2(b^2 + 2*a*c)
>Que e o que voce deseja demonstrar.


Ué, mas a desigualdade a ser mostrada é
1<=c/raiz(c^2 + 8*a*b)+ a/raiz(a^2 + 8*b*c)+ b/raiz(b^2 + 8*a*c)
e não
1<=c/raiz(c^2 + 2*a*b)+ a/raiz(a^2 + 2*b*c)+ b/raiz(b^2 + 2*a*c)...

(mas a demonstração para esta última foi bonita..)

Bruno

>Eu estou com pouco tempo, mas, atendendo o apelo do Prof Nicolau e do
Colega
>Marcelo Rufino, que gostariam de ver solucoes diferentes das que sao
>apresentadas no Site onde publicam as solucoes das questoes IMO, bolei,
>neste ultimo domingo (ontem) duas outras maneiras diferentes de fazer
aquela
>questao que a lenda diz ser a mais dificil de todas as IMOS :
>
>Se (a^2 + b^2)/(a*b + 1) e um inteiro, entao este inteiro e um quadrado
>perfeito.
>
>1) A primeira e geometrica, interpretando "a" e "b" como catetos de um
>triangulo retangulo
>2) A segunda e por progressoes geometricas, interpretando (a^2 + b^2)/(a*b
+
>1) como
>
>(a^2 + b^2)/(a*b + 1)= a^2/(ab+1)  +  b^2/(ab + 1)
>(a^2 + b^2)/(a*b + 1)= 1/(b/a + 1/a^2) +  1/(a/b + 1/b^2)
>
>Em verdade, ha muito tempo atras, eu e o nosso muito estimado colega Bruno
>Leite, membro desta lista, ( que ha muito nao escreve para a lista e anda
>desaparecido) encontramo-nos num Chat e passamos a discutir sobre esta
>questao, quando entao o Bruno me apresentou uma  solucao "tipo resolucao de
>equacao do 2 grau" e eu apresentei as duas solucoes a que me referi acima.
>
>Eu nao guardo as solucoes que faco, de forma que tive que redescobrir meu
>raciocinio para dar este presente a voces. Logo, logo, quando  nos proximos
>dias eu tiver um tempo livre, vou publicar aqui estas solucoes.
>
>De coracao,
>
>Um abraco a todos
>Paulo Santa Rita
>2,1657,09072001




Re: [imo-problems] IMO 2001 Problems, First Day

2001-07-08 Por tôpico Bruno Leite

Sobre o segundo problema:

Se (a,b,c) satisfaz a/sqrt(a^2+8bc) + b/sqrt(b^2+8ca) + c/sqrt(c^2+8ab) >= 1
ent?o (ka,kb,kc) (k>0) também satisfaz. Ent?o podemos supor que abc=1/8.

Ent?o ficamos com 1/sqrt(1+8bc/a^2)+1/sqrt(1+8ac/b^2)+1/sqrt(1+8ab/c^2)>=1
ou 1/sqrt(1+8abc/a^3)+1/sqrt(1+8abc/b^3)+1/sqrt(1+8abc/c^3)>=1 ou
1/sqrt(1+8/a^3)+1/sqrt(1+8/b^3)+1/sqrt(1+8/c^3)>=1 com abc=1/8.

Veja que se 1/a^3=A, 1/b^3=B e 1/c^3=C temos ABC=512 e temos que provar que

1/sqrt(1+A)+1/sqrt(1+B)+1/sqrt(1+C)>=1 se ABC=512 (**)

Eu consegui provar um resultado parecido só que mais fraco:

1/(1+A)+1/(1+B)+1/(1+C)>=1/3 se ABC=512

Isso n?o implica (**) pois ( sqrt(x)+sqrt(y)+sqrt(z) )^2 =< 3*(x+y+z), para
quaisquer x, y, z >0.

Bom, eu nem sei se estou no caminho certo, mas, mesmo assim, alguém tem
alguma idéia para provar (**)?

Bruno Leite

-Mensagem original-
De: Marcio <[EMAIL PROTECTED]>
Para: [EMAIL PROTECTED] <[EMAIL PROTECTED]>
Data: Domingo, 8 de Julho de 2001 18:28
Assunto: ENC: [imo-problems] IMO 2001 Problems, First Day


>Acabei de receber isso em outra lista.. Agora vou tentar fazer
>(provavelmente sem sucesso :) e depois mandar meus comentarios pra gente
>debater na lista!
>Abracos,
>Marcio
>
>-Mensagem original-
>De: Mojca Miklavec [mailto:[EMAIL PROTECTED]]
>Enviada em: Domingo, 8 de Julho de 2001 15:41
>Para: [EMAIL PROTECTED]
>Assunto: [imo-problems] IMO 2001 Problems, First Day
>
>
>
>1. Let ABC be an acute-angled triangle with circumcentre O. Let P on BC be
>the foot of the altitude from A.
>
>Suppose that <)BCA >= <)ABC + 30°.
>
>Prove that <)CAB + <)COP < 90°.
>
>
>2. Prove that
>
>a/sqrt(a^2+8bc) + b/sqrt(b^2+8ca) + c/sqrt(c^2+8ab) >= 1
>
>for all positive real numbers a, b and c.
>
>
>3. Twenty-one girls and twenty-one boys took part in a mathematical
>contest.
>
>- Each contestant solved at most six problems.
>- For each girl and each boy, at leat one problem was solved by both of
>them.
>
>Prove that there was a problem that was solved by at least three girls and
>at leat three boys.
>
>
>imo-problems is part of the IMO network, http://imonet.online.fr/
>remember to go there and add your name !
>
>To receive these messages in a daily digest, email
>[EMAIL PROTECTED]
>To stop recieving these messages, email [EMAIL PROTECTED]
>You can always view and reply to messages on-line, at
>http://www.egroups.com/group/imo-problems
>For more help, email [EMAIL PROTECTED]
>
>




Re: problema

2001-07-05 Por tôpico Bruno Leite

H...

As soluções do http://www.kalva.demon.co.uk/ são muito compactas, né? Acho
que não valem nada se vc não pensou no problema...

Bruno Leite

PS Acho que a solução que eu mostrei é mais ou menos a mesma que está lá no
site dele.

-Mensagem original-
De: Marcelo Rufino de Oliveira <[EMAIL PROTECTED]>
Para: [EMAIL PROTECTED] <[EMAIL PROTECTED]>
Data: Sexta-feira, 6 de Julho de 2001 01:21
Assunto: Re: problema


>Um comentário com relação as questões propostas das imos pelo Henrique é
>que, apesar de vários de nossos colegas terem capacidade de resolvê-las e
>acrescentar bastante à lista, a discussão de problemas antigos das imo
>perdeu um pouco a "graça" depois que lançaram um site que contem todas as
>imo resolvidas, desde a primeira, em 1959, até a última, em 2000. O
endereço
>é http://www.kalva.demon.co.uk/ e é de autoria de um inglês (obviamente a
>página é toda em inglês), inclusive tem um link no site da obm para este
>site. Nesta página tem também as Putnam resolvidas desde 1975, certamente
>para quem nunca viu vale a pena dar uma olhada. A segunda questão proposta
é
>bem famosa e é encarada como uma das mais difíceis que já cairam em imos.
Eu
>já devo ter visto pelo menos umas 5 soluções distintas para esta questão em
>vários livros de olimpíadas (Winning Solutions, Mathematical Olympiad
>Challenges, etc.).
>De toda maneira, soluções distintas das apresentadas no site que eu citei
>são bem vindas.
>
>Falou,
>Marcelo Rufino
>
>- Original Message -
>From: Henrique Lima Santana <[EMAIL PROTECTED]>
>To: <[EMAIL PROTECTED]>
>Sent: Thursday, July 05, 2001 10:53 PM
>Subject: Re: problema
>
>
>>
>>
>>
>>
>>
>>
>>  Valeu Paulo! Essas não eram realmente difíceis, mas esse seu método
>de
>> ensinar até q é legal hehe :)
>>  Agora tem 2 aqui q são bem difíceis(aparentemente), pelo menos eu
não
>> consegui sair do lugar:
>> 1. (imo 90) determine todos os n naturais tais q ( 2^n +1 )/n^2 é
>> inteiro
>> 2. (imo 88) prove q se a e b são naturais e (a^2 + b^2)/(ab + 1) é
>> inteiro então (a^2 + b^2)/(ab + 1) é quadrado perfeito
>>  Obrigado mais uma vez,
>>   []´s Henrique
>>
>>
>>
>>
>>
>>
>>
>>
>>
>>
>>
>>
>>
>>
>>
>>
>> >From: "Paulo Santa Rita" <[EMAIL PROTECTED]>
>> >Reply-To: [EMAIL PROTECTED]
>> >To: [EMAIL PROTECTED]
>> >Subject: Re: problema
>> >Date: Thu, 05 Jul 2001 19:13:04
>> >
>> >Ola Henrique,
>> >
>> >Sem duvida que voce pode fazer as questoes abaixo ...
>> >
>> >>From: "Henrique Lima Santana" <[EMAIL PROTECTED]>
>> >>Reply-To: [EMAIL PROTECTED]
>> >>To: [EMAIL PROTECTED]
>> >>Subject: problema
>> >>Date: Thu, 05 Jul 2001 14:18:53 -0300
>> >>
>> >>olah, algumas duhvidas nessas questões de teoria dos nºs...
>> >>
>> >>1.prove q pra todo n natural, 3^2n+1 + 2^n+2 é multiplo de 7 e que
>> >>3^2n+2 + 2^6n+1 é multiplo de 11.
>> >
>> >
>> >3^(2n+1) + 2^(n+2)=3*(9^n) + 4*(2^n)
>> >Para n=1 => 3*(9^n) + 4*(2^n)= 35 ( divisivel por 7 )
>> >
>> >Ja que 7 | 3*(9^n) + 4*(2^n) entao 7 | 2*( 3*(9^n) + 4*(2^n) )
>> >entao 7 | 6*(9^n) + 8*(2^n) entao 7 | ( 6*(9^n) + 8*(2^n) ) + 21*9^n
>> >entao 7 | 27*(9^n) + 8*(2^n) entao 7 |3*(9^(n+1)) + 4*(2^(n+1))
>> >
>> >Logo, vale para todo n natural.
>> >Agora voce faz o caso 11, falou ?
>> >
>> >
>> >>2.mostrar q pra nenhum n natural 2^n + 1 é um cubo.
>> >
>> >2^n + 1=a^3 => "a^3" e impar => "a" e impar
>> >2^n = a^3 - 1 => 2^n=(a-1)*(a^2 + a + 1)
>> >Nao e essa ultima igualdade um evidente absurdo ? (Por que ?)
>> >Entao, 2^n + 1 nao pode ser um cubo perfeito !
>> >
>> >
>> >>3. mostrar q 3 eh o unico primo p / p, p+2 e p+4 são todos primos.
>> >
>> >olhe para p,p+1,p+2,p+3,p+4. Dado que "p" e primo entao ele deixa resto
1
>> >ou
>> >2 ( e congruo a ) quando dividido por 3, certo ? E entao:
>> >
>> >se o resto for 1 implica o que ?
>> >se o resto for 2 implica o que ?
>> >
>> >
>> >
>> >>  qualquer ajuda será bem-vinda!
>> >>  Henrique
>>
>>>_
>> >>Get Your Private, Free E-mail from MSN Hotmail at
>http://www.hotmail.com.
>> >
>> >Francamente, depois destas posso avaliar como deve se sentir um desses
>Tios
>> >de Jardim de Infancia ... Beeen ! (A mumia paralitica toca o sinete )
>> >Valeu.
>> >E isso ai Henrique. Cai dentro que Matematica e como andar de bicicleta
:
>> >so
>> >fazendo muitos exercicios a gente se desenrola e adquire desenvoltura.
>> >
>> >Um abraco pra voce
>> >Paulo Santa Rita
>> >5,1612,05072001
>> >
>>
>_
>> >Get Your Private, Free E-mail from MSN Hotmail at
http://www.hotmail.com.
>> >
>>
>> _
>> Get Your Private, Free E-mail from MSN Hotmail at http://www.hotmail.com.
>>
>>
>




Re: problema

2001-07-05 Por tôpico Bruno Leite


-Mensagem original-
De: Henrique Lima Santana <[EMAIL PROTECTED]>
Para: [EMAIL PROTECTED] <[EMAIL PROTECTED]>
Data: Quinta-feira, 5 de Julho de 2001 23:02
Assunto: Re: problema


>
>
>
>
>
>
> Valeu Paulo! Essas não eram realmente difíceis, mas esse seu método de
>ensinar até q é legal hehe :)
> Agora tem 2 aqui q são bem difíceis(aparentemente), pelo menos eu não
>consegui sair do lugar:
>1. (imo 90) determine todos os n naturais tais q ( 2^n +1 )/n^2 é
>inteiro
>2. (imo 88) prove q se a e b são naturais e (a^2 + b^2)/(ab + 1) é
>inteiro então (a^2 + b^2)/(ab + 1) é quadrado perfeito

O segundo problema é, segundo a lenda, o mais difícil de todas as imos.

Eu conheço uma solução curta mas com uma idéia bem "do outro mundo".

Vamos dizer que (x,y) é uma solução se x e y são inteiros e (x^2 +
y^2)/(xy+1) é inteiro.

Se (a,b) é uma solução, então (a^2 + b^2)/(ab + 1)=k, com k inteiro. Então
temos a^2 + b^2 -kab - k=0. Vamos mostrar que k é um quadrado perfeito.

Considere a equação de 2º grau em x  x^2 + b^2 -kbx - k=0. Quais são suas
soluçoes? Uma é "a" (óbvio!) e a outra é "kb-a", pois a soma das soluções
deve dar kb. Então se (a,b) é uma solução com a>b>=0 teremos que (b, kb-a)
também é uma solução.

Suponha que já conseguimos provar que b>kb-a>=0. Se chamarmos kb-a de c,
vimos que se (a,b) é solução com a>b>=0 então (b, c) também será solução com
b>c>=0. repetindo o processo e chamando d=kc-b, vemos que (c, d) será
solução com c>d>=0.

Isso mostra que se acharmos uma solução (a_1,a_2), então podemos achar
outras soluções (a_2,a_3), (a_3,a_4),etc, com a_1>a_2>a_3>a_4...  Como
a_1,a_2,a_3,a_4... são inteiros não-negativos, não podem decrescer sempre:
haverá uma hora em que teremos a_j=0. Assim como (a_2,a_3), (a_3,a_4),etc
 são soluções, (a_(j-1),a_j) é solução. Isso significa que [a_(j-1)^2 +
(a_j)^2] / [a_(j-1)*a_j + 1] = k, ou seja,
k=a_(j-1)^2. Logo k é um quadrado perfeito.

Veja que só ficou faltando a passagem "a>b>0 Implica b>kb-a>0". Não é muito
difícil, vou deixar para vocês completarem...é o sono...

Espero não ter escrito muitas bobagens, amanhã vou ler direitinho o que eu
escrevi..

Bruno Leite

> Obrigado mais uma vez,
>  []´s Henrique





Re: encontre o erro

2001-07-05 Por tôpico Bruno Leite


-Mensagem original-
De: Henrique Lima Santana <[EMAIL PROTECTED]>
Para: [EMAIL PROTECTED] <[EMAIL PROTECTED]>
Data: Quinta-feira, 5 de Julho de 2001 23:21
Assunto: encontre o erro


>
>
>   9 - 21 = 16 - 28
>  somando 49/4 a cada membro temos
>9 - 21 + 49/4 = 16 - 28 + 49/4 <=> 3^2 - 2*3*7/2 + (7/2)^2= 4^2 -   -
>2*4*7/2 + (7/2)^2 <=> (3 - 7/2)^2 = (4 - 7/2)^2
>   simplificando os quadrados

Não pode fazer isso!

a^2 = b^2 NÃO IMPLICA a=b. A função x^2 não é injetora...

O certo é a^2=b^2 -> a^2-b^2=0 -> (a-b)(a+b)=0 -> a=+-b

>  3 - 7/2 = 4 - 7/2
>somando 7/2 aos 2 membros temos
>3=4
>
>
>
>
>
>
>
>
>
>
>
>
>
>
>
>
>>From: "Paulo Santa Rita" <[EMAIL PROTECTED]>
>>Reply-To: [EMAIL PROTECTED]
>>To: [EMAIL PROTECTED]
>>Subject: Re: problema
>>Date: Thu, 05 Jul 2001 19:13:04
>>
>>Ola Henrique,
>>
>>Sem duvida que voce pode fazer as questoes abaixo ...
>>
>>>From: "Henrique Lima Santana" <[EMAIL PROTECTED]>
>>>Reply-To: [EMAIL PROTECTED]
>>>To: [EMAIL PROTECTED]
>>>Subject: problema
>>>Date: Thu, 05 Jul 2001 14:18:53 -0300
>>>
>>>olah, algumas duhvidas nessas questões de teoria dos nºs...
>>>
>>>1.prove q pra todo n natural, 3^2n+1 + 2^n+2 é multiplo de 7 e que
>>>3^2n+2 + 2^6n+1 é multiplo de 11.
>>
>>
>>3^(2n+1) + 2^(n+2)=3*(9^n) + 4*(2^n)
>>Para n=1 => 3*(9^n) + 4*(2^n)= 35 ( divisivel por 7 )
>>
>>Ja que 7 | 3*(9^n) + 4*(2^n) entao 7 | 2*( 3*(9^n) + 4*(2^n) )
>>entao 7 | 6*(9^n) + 8*(2^n) entao 7 | ( 6*(9^n) + 8*(2^n) ) + 21*9^n
>>entao 7 | 27*(9^n) + 8*(2^n) entao 7 |3*(9^(n+1)) + 4*(2^(n+1))
>>
>>Logo, vale para todo n natural.
>>Agora voce faz o caso 11, falou ?
>>
>>
>>>2.mostrar q pra nenhum n natural 2^n + 1 é um cubo.
>>
>>2^n + 1=a^3 => "a^3" e impar => "a" e impar
>>2^n = a^3 - 1 => 2^n=(a-1)*(a^2 + a + 1)
>>Nao e essa ultima igualdade um evidente absurdo ? (Por que ?)
>>Entao, 2^n + 1 nao pode ser um cubo perfeito !
>>
>>
>>>3. mostrar q 3 eh o unico primo p / p, p+2 e p+4 são todos primos.
>>
>>olhe para p,p+1,p+2,p+3,p+4. Dado que "p" e primo entao ele deixa resto 1
>>ou
>>2 ( e congruo a ) quando dividido por 3, certo ? E entao:
>>
>>se o resto for 1 implica o que ?
>>se o resto for 2 implica o que ?
>>
>>
>>
>>>  qualquer ajuda será bem-vinda!
>>>  Henrique
>>>_
>>>Get Your Private, Free E-mail from MSN Hotmail at http://www.hotmail.com.
>>
>>Francamente, depois destas posso avaliar como deve se sentir um desses
Tios
>>de Jardim de Infancia ... Beeen ! (A mumia paralitica toca o sinete )
>>Valeu.
>>E isso ai Henrique. Cai dentro que Matematica e como andar de bicicleta :
>>so
>>fazendo muitos exercicios a gente se desenrola e adquire desenvoltura.
>>
>>Um abraco pra voce
>>Paulo Santa Rita
>>5,1612,05072001
>>
>>_
>>Get Your Private, Free E-mail from MSN Hotmail at http://www.hotmail.com.
>>
>
>_
>Get Your Private, Free E-mail from MSN Hotmail at http://www.hotmail.com.
>




Re: Nivel Universitario.

2001-06-22 Por tôpico Bruno Leite

Olha só

-Mensagem original-
De: Olimpiada Brasileira de Matematica <[EMAIL PROTECTED]>
Para: [EMAIL PROTECTED] <[EMAIL PROTECTED]>
Data: Sexta-feira, 22 de Junho de 2001 16:09
Assunto: Nivel Universitario.


>Caros amigos da lista:
>
>-Informacoes para o Nivel Universitario.
>-Lista de Universidades Cadastradas ate o momento.
>
>http://www.obm.org.br/nivelu.htm
>
>Abracos,
>
>Nelly.
>




Re: Somatório !

2001-05-28 Por tôpico Bruno Leite

ok, é que vc falou que tinha _quase_ certeza que convergia...
bom, eu acho que não converge para nenhum número especial, então calculei
numericamente mesmo.

-Mensagem original-
De: Rodrigo Villard Milet <[EMAIL PROTECTED]>
Para: [EMAIL PROTECTED] <[EMAIL PROTECTED]>
Data: Segunda-feira, 28 de Maio de 2001 10:22
Assunto: Re: Somatório !


>Me desculpe, mas na segunda questão, a pergunta era pra qt converge a
série.
>¡Villard!
>-Mensagem original-
>De: Bruno Leite <[EMAIL PROTECTED]>
>Para: [EMAIL PROTECTED] <[EMAIL PROTECTED]>
>Data: Domingo, 27 de Maio de 2001 02:16
>Assunto: Re: Somatório !
>
>
>>Usando computador: o somatório de 1/k^k converge para
>>1.291285997062663540407282590595600541498619368274522317310002445136944538
7
>6
>>523448817041129429709...
>> Dá para provar que converge, sem computador: se você comparar os termos
da
>>sua série com alguma série geométrica fica fácil. Explicando melhor: Para
>>k>2, 0<1/k^k< 1/2^k, e como a soma até infinito de 1/2^k converge, a soma
>>até infinito de 1/k^k deve convergir.
>>
>>Em relação à primeira questão, acho (eu ACHO!) que não tem forma
>fechada.(eu
>>ACHO)
>>
>>Bruno Leite
>>-Mensagem original-
>>De: Rodrigo Villard Milet <[EMAIL PROTECTED]>
>>Para: Obm <[EMAIL PROTECTED]>
>>Data: Domingo, 27 de Maio de 2001 01:14
>>Assunto: Somatório !
>>
>>
>>1) É possível calcular o somatório de k^k, com k variando de 1 até n
??
>>2) O somatório de (1/k)^k, com k variando de 1 até infinito converge
??
>>pra qt ?
>>Tenho quase certeza de q ela converge,. mas ñsei pra qt...
>>¡Villard!
>>
>>
>>
>




Re: Somatório !

2001-05-26 Por tôpico Bruno Leite

Usando computador: o somatório de 1/k^k converge para
1.29128599706266354040728259059560054149861936827452231731000244513694453876
523448817041129429709...
 Dá para provar que converge, sem computador: se você comparar os termos da
sua série com alguma série geométrica fica fácil. Explicando melhor: Para
k>2, 0<1/k^k< 1/2^k, e como a soma até infinito de 1/2^k converge, a soma
até infinito de 1/k^k deve convergir.

Em relação à primeira questão, acho (eu ACHO!) que não tem forma fechada.(eu
ACHO!!!!)

Bruno Leite
-Mensagem original-
De: Rodrigo Villard Milet <[EMAIL PROTECTED]>
Para: Obm <[EMAIL PROTECTED]>
Data: Domingo, 27 de Maio de 2001 01:14
Assunto: Somatório !


1) É possível calcular o somatório de k^k, com k variando de 1 até n ??
2) O somatório de (1/k)^k, com k variando de 1 até infinito converge ??
pra qt ?
Tenho quase certeza de q ela converge,. mas ñsei pra qt...
¡Villard!





Re: problema de probabilidade...

2001-05-22 Por tôpico Bruno Leite

Isso é da OBM 99, terceiro nível, segunda fase! A solução deve estar em
alguma eureka!!

Bruno Leite

PS Não abram o attachment que for junto com o email; acho que o meu
computador está com vírus!
-Mensagem original-
De: Luis Lopes <[EMAIL PROTECTED]>
Para: [EMAIL PROTECTED] <[EMAIL PROTECTED]>
Data: Terça-feira, 22 de Maio de 2001 19:17
Assunto: problema de probabilidade...


Sauda,c~oes,

Repasso um problema de uma outra lista.

[ ]'s
Lu'is


>From: "Daniel Cid (sinistrow)" <[EMAIL PROTECTED]>
>Reply-To: [EMAIL PROTECTED]
>To: [EMAIL PROTECTED]
>Subject: [Olympium] problema de probabilidade...
>Date: Fri, 18 May 2001 13:31:27 -0300
>
> Alguem pode me ajudar nesse problema ??
>
>Jose tem tres pares de oculos, um marrom, um amarelo e um cinza. Todo
>dia ele escolhe um ao acaso, tendo apenas o cuidado de nunca usar o
>mesmo que usou no dia anterior. Se dia primeiro de agosto ele usou o
marrom.
>Qual a probabilidade de que no dia 31 de agosto ele volte a usar o
marrom ???
>
>[]`z
>
>--
>Daniel B. Cid (sinistrow)





Re: Alguém pode dar uma explicação em algumas dúvidas?

2000-10-19 Por tôpico Bruno Leite

>2)Como posso provar fi de Euler: a^[fi(n)] == 1 (mod n)? Eu vi a prova no
>Eureka 2, porém não a entendi por completo, pois há algumas transformações
>que me pareceram "mágicas". :)

Como assim, provar fi de Euler? Acho que vc quer ver como se prova que
sendo fi(n) a função de Euler, então a^[fi(n)] == 1 (mod n) SE a É PRIMO
COM n...

Então sejam x(i) os números menores que n e primos com n. Existem
exatamente fi(n) desses números. Escolha um "a" primo com n. Então os
números a*x(1), a*x(2), ..., a*x(fi(n)) são todos primos com n(pois "a" é
primo com n e os a(i) são primos com n) e além disso eles são todos
distintos módulo n.( se a*x(i)==a*x(j) (mod n) então x(i)==x(j) (mod n)
porque "a"é primo com n, o que permite que ele seja "cancelado" na
congruência)

Então, mod n, os conjuntos R={x(i)} e S={a*x(i)} são iguais(os dois são o
conjunto dos números menores que n e primos com n) Logo o produto de todos
os seus elementos são iguais (sempre mod n)

Produtório dos elementos de R==Produtório dos elementos de S
Produtório(i de 1 a fi(n)) de x(i) == Produtório(i de 1 a fi(n)) de a*x(i)
Produtório(i de 1 a fi(n)) de x(i)== a^fi(n) * Produtório(i de 1 a fi(n))de
x(i)

E como os produtórios cancelam temos a^fi(n)==1(mod n)

Espero ter ajudado um pouco.

3) Problema:
>
>Prove que dado n pertence N existe um conjunto de n  elementos   A está
>contido em N tal que para todo B está contido em A, B diferente de vazio, a
>somatória de x talque x pertence a B é uma potência não trivial (isto é, um
>número da forma m^k, onde m, k  são inteiros maiores ou iguais a 2), ou
>seja, A = {x_1, x_2,. x_n} tal que x_1, x_2,.x_n, x_1 + x_2, x_1 +x_3,., ,
>.,x_1 + x_2 +.x_n  são todos potências não triviais.
>
Eu acho que esse enunciado está um pouco confuso...vc poderia reformular
por favor?

Bruno Leite




Re: a parabola eh legal

2000-10-14 Por tôpico Bruno Leite



(etc)
>
>Ah, outra propriedade da cicloide é ela também ser a curva brachistócrona,
>ou seja, aquela que liga dois pontos A e B não pertencentes a mesma vertical
>e a mesma horizontal de tal forma que o tempo para ir de A até B,
>percorrendo a curva é mínimo.

Como assim?

Bruno Leite

>Até mais...
>
>
>
>> ~~
>> Rui L Viana F
>> [EMAIL PROTECTED]
>> [EMAIL PROTECTED]
>> [EMAIL PROTECTED]
>> ~~
>
>Ei, como é o MIT? Que curso voce está fazendo aí?
>
>
>
>




Recorrência

2000-10-06 Por tôpico Bruno Leite

>X-Originating-IP: [200.224.111.4]
>From: "Marcelo Souza" <[EMAIL PROTECTED]>
>To: [EMAIL PROTECTED]
>Subject: Recorrência
>Date: Fri, 06 Oct 2000 17:01:59 GMT
>X-OriginalArrivalTime: 06 Oct 2000 17:01:59.0309 (UTC)
FILETIME=[23D583D0:01C02FB7]
>Sender: [EMAIL PROTECTED]
>Reply-To: [EMAIL PROTECTED]
>
>Olá pessoal! Alguém poderia me explicar detalhadamente o exercício a seguir?
>
>. Defina, por recorrência, uma função f:N->N estipulando que f(1)=3 e
>f(n+1)=5.f(1)+1. Dê uma forma explícita para f(n).

OI Marcelo

f(n+1)=5.f(n)+1
f(1)=3
Isso aí é quase uma PG.

Temos f(n+1) + 1/4 =5.f(n)+1 +1/4
Logo f(n+1) + 1/4 =5.(f(n) +1/4)

Chame f(n)+1/4 de h(n)

temos h(n+1)=5h(n) e h(1)=3+1/4=13/4

Então h(n)=5^(n-1) * 13/4 e f(n)=5^(n-1) * 13/4 -1/4

Está tudo certo?

Bruno Leite

>Obrigado
>Abraços
>Marcelo





Re: Ajuda

2000-10-02 Por tôpico Bruno Leite

At 19:02 02/10/00 EDT, you wrote:
>
>   Olá.
>   Por favor, me ajudem nesse exercício:
>
>  (FATEC)-Seja n pertencente IN, tal que
>
>   (1+2+3+...+n)/(n+1)! = 1/240

Entao n(n+1) / 2(n+1)! = 1/240 -> 1/2(n-1)! = 1/240 -> (n-1)!=120 -> n=6...

Bruno Leite
>
>   Então o valor de log  (2n+4) é:
>   1/2
>
>  a)-4   b)4   c)-2sqrt2   d)2sqrt2e)1/2
>
>
>Muito obrigado.
>
>
>
>
>João Paulo Paterniani da Silva
>
>_
>Get Your Private, Free E-mail from MSN Hotmail at http://www.hotmail.com.
>
>Share information about yourself, create your own public profile at 
>http://profiles.msn.com.
>
>




Ibero

2000-09-25 Por tôpico Bruno Leite

Onde posso achar a prova da Ibero desse ano?




Re: vale o "raciocínio"

2000-09-24 Por tôpico Bruno Leite

At 16:02 23/09/00 -0300, you wrote:
>Amigos listeiros...
>alguns já se inscreveream na ciencialist?? Espero que sim, a conversa lá
>está boa. Para os que perderam, é uma lista de ciências da página do eGroups
>(www.egroups.com).
>
>Vendo que foi usada uma notação de limite na lista, resolvi fazer a minha
>linha pouco lógica de raciocínio.
>Ora:
>lim (1 + 1/x)^x = lim (1 + 0)^(oo) = lim 1 ^ (oo)
>x -> oo   x -> oo   x -> oo
>
>Portanto 1 elevado na infinito não pode ser definido. Está correto esse
>raciocínio?

É verdade, 1 elevado a infinito é indefinido.

Pode dar 1 (lim 1^x para x tendendo a infinito)
Pode dar e=2,71828...
Pode "dar" infinito : lim[ (1 + sqrt(1/x) ) ^ (x) ]para x tendendo a
infinito dá infinito.
Pode dar qualquer coisa. Por exemplo, lim( (1+k/x)^x ) para x tendendo a
infinito dá e^k

Bruno Leite

>Ou seria 1 diferente de 1,0...001 (= lim(1/x) com x -> oo)
>
>Abraço,
>
>
>Benjamin Hinrichs
>
>
>




Re: protesto

2000-09-18 Por tôpico Bruno Leite

>Perdoem-me se esta mensagem não se enquadra perfeitamente com o perfil
desta lista, mas achei seu conteúdo tão interessante, importante e
divertido >quanto alguns problemas.
  >
>(Peço a especial atenção dos PROFESSORES).
>
>REVOLTADO OU CRIATIVO ?!
>
>De Waldemar Setzer, professor aposentando da USP
>(...)

O professor Setzer tem uma HP que tem mais material do tipo:
www.ime.usp.br/~vwsetzer
Inclusive lá tem a história do barômetro.

Nós do ime-usp tivemos a sorte de assistir a uma palestra dele durante a
semana de recepção dos calouros, quando ele falou sobre a teoria de cores
de Goethe. (aliás isso também deve estar na HP)

Bruno




Re: Questão interessante

2000-09-11 Por tôpico Bruno Leite

At 19:40 10/09/00 -0300, you wrote:
>Olá, pessoal, venho aqui trazer uma  questão que eu
>inventei enquanto rabiscava num papel aqui em casa...  é o
>seguinte. Provar : <  [somatório de k^(1/k) com k de 2
>até< 2n -  ln(n+1) . 

Quem é esse n?

Bruno Leite

É bastante
>interessante a  resolução que eu dei se alguém
>quiser eu mando, mas por  enquanto, tentem ! Abraços, 
>!Villard! 




Re: Bibliografia / Cálculo

2000-09-04 Por tôpico Bruno Leite

At 20:02 04/09/00 -0300, you wrote:
>   
> Olá
>
>   Gostaria de saber se alguém poderia me recomendar alguns livros sobre 
>matemática/física. Sobre os segundos, algúem sabe se os da coleção LPM Maia 
>são bons? Quanto aos de matemática, tenho a coleção do Gelson Iezzi, mas 
>gostaria de algo mais introdutório também, noções básicas de geometrica, 
>cálculo de áreas de figuras planas e métodos de cálculo (racionalização de 
>frações, subtração membro a membro, etc), principalmente em relação aos 
>últimos, que bibliografia poderia consultar?
>
Sobre as noções básicas de Geometria você deveria procurar algo no livro
dos professores Wagner e Morgado.(que são da lista) É um livro meio chato
de se achar, mas talvez vc encontre em sebos ou em bibliotecas de
faculdades. Eu sei que tem um exemplar na UNICAMP. É um otimo livro...

Sobre os "métodos de cálculo", eu acho que talvez o livro das olimpíadas
russas seja o ideal pois ele tem um arsenal de truques e "idéias espertas".
Na Amazon.com você pode comprá-lo por uns 10 ou 11 dólares. Tem também um
livro do Lidski, que tem exercícios de dificuldade intermediária. (mais
difícil que os vestibulares e mais fácil que as olimpíadas )

Espero ter ajudado.

Bruno

>Grato
>
>
>
>
>"Against stupidity, the Gods themselves contend in vain",
> Friedrich von Schiller's
>-
>[]'s
>{O-Grande-Mentecapto}
>[EMAIL PROTECTED]
>
>
>




IMO2000- problema2

2000-08-25 Por tôpico Bruno Leite

IMO2000-Problema 2
a,b,c sao reais positivos e abc=1.

Mostre que (a - 1 + 1/b)(b - 1 + 1/c)(c - 1 + 1/a)<=1.

O Eduardo Tengan me deu a seguinte dica:
Faca a=u/v, b=v/w, c=w/u, com u,v,w>0. Ai' foi *muito* facil
completar...vou ate deixar um espaco para quem nao quiser olhar.


















(espaco)
















Substituindo temos:

(u/v - 1 + w/v)(v/w - 1 + u/w)(w/u - 1 + v/u)<=1
ou (u - v + w)(v - w + u)(w - u + v)<=uvw (***)

Chamando (u+v+w)=2p, temos (2p-2v)(2p-2w)(2p-2u)<=uvw
ou ainda (p-v)(p-w)(p-u)<=uvw/8

CASO 1)Se u,v,w sao lados de um triangulo, temos
p(p-v)(p-w)(p-u)/R <= (p/2)(uvw/4R) [R eh o raio da circ.circunscrita]
(Area)^2 /R <= (p/2)(Area)
(Area)r/R <= pr/2 = (area)/2 [R eh o raio da circ.inscrita]
r/R<=1/2 ou 2r<=R, o que é verdade para qualquer triangulo (isso eh
consequencia de um teorema de Euler)

CASO 2) Se u,v,w nao formam triangulo, temos u>=v+w ou v>=u+w ou w>=u+v

Eh facil ver que o lado esquerdo de (***) eh negativo ou nulo, e portanto
claramente menor que uvw, que eh positivo.

Como de costume, pode haver algum erro na solucao e se alguem achar eu
gostaria que avisasse para a lista.
Ou ainda, ela pode estar _completamente_ errada, como aconteceu com a minha
"solucao" do problema 5...
Aliás eu fiquei bem surpreso com a solucao do Gugu pois eu( e alias varios
colegas do IME-USP tambem) pensava que realmente nao haveria um numero
assim

Bruno Leite






Re: IMO-2000 --- problema 5

2000-08-11 Por tôpico Bruno Leite

At 22:51 10/08/00 -0300, you wrote:
>
>Oi,Bruno,
>  A sua solucao tem um problema:q-1 de fato nao pode dividir 3
>elevado a k,mas nao precisa ser primo com 3,e portanto voce nao pode
>concluir que q-1 divide 2u.
>  Abracos,
>Gugu

Nossa, é mesmoe nem dá pra consertar....

Bom, valeu.. :-)

Bruno Leite

>P.S.:Oi,Nicolau,manda a minha solucao xroteada...
>>
>>Oi pessoal
>>
>>Agora estou enviando a minha solucao do quinto problema da IMO-2000.
>>Corrijam-me se acharem alguma besteira...
>>
>>Vou usar o sinal de igual para congruência.
>>
>>Temos que 2^n+1 é multiplo de n e n é divisível por 2000 primos.
>>Seja k o maior inteiro tal que 3^k divide n. Seja q o menor primo(fora o 3)
>>que divide n (logo q>3). (ou ainda: q é o menor primo que divide n/(3^k)...
>>) E seja P=n/((3^k)*q). Veja que P é um produto de milhares de primos,
>>contendo talvez uma potência de q em sua fatoração. Mas veja que qualquer
>>primo de P é maior ou igual a q.
>>
>>Entao n=q*P*3^k.
>>Temos 2^n=-1(mod n) e portanto 2^n=-1(mod q)
>>2^(q*P*3^k)=-1(mod q) ->pelo pequeno teorema de Fermat ->  (2^(P*3^k))^q =
>>-1 = 2^(P*3^k) (mod q)
>>Logo
>>
>> 2^(2*P*3^k) = 1 (mod q)
>>
>>Seja r uma raiz primitiva mod q e seja u tal que r^u=2 (mod q), com 0>Então r^(2*P*u*3^k) = 1 (mod q)
>>
>>Por isso q-1 divide 2*P*u*3^k
>>
>>Mas q-1 não divide 3^k pois q-1 é par. E q-1 não divide P pois q-1 é menor
>>que qualquer primo de P.
>>
>>Logo q-1 divide 2*u com 0>Por isso u=q-1 ou u=(q-1)/2
>>
>>Como r^u=2(mod q), se u for q-1 teremos 1=2 mod q absurdo. Entao u=(q-1)/2
>>Mas aí r^u=+-1(mod q) ->q=1(absurdo) ou q=3(absurdo por definicao de q)
>>
>>Portanto não existe um tal n.
>>---
>>
>>Bruno Leite
>>
>>
>>
>
>




IMO-2000 --- problema 5

2000-08-10 Por tôpico Bruno Leite

Oi pessoal

Agora estou enviando a minha solucao do quinto problema da IMO-2000.
Corrijam-me se acharem alguma besteira...

Vou usar o sinal de igual para congruência.

Temos que 2^n+1 é multiplo de n e n é divisível por 2000 primos.
Seja k o maior inteiro tal que 3^k divide n. Seja q o menor primo(fora o 3)
que divide n (logo q>3). (ou ainda: q é o menor primo que divide n/(3^k)...
) E seja P=n/((3^k)*q). Veja que P é um produto de milhares de primos,
contendo talvez uma potência de q em sua fatoração. Mas veja que qualquer
primo de P é maior ou igual a q.

Entao n=q*P*3^k.
Temos 2^n=-1(mod n) e portanto 2^n=-1(mod q)
2^(q*P*3^k)=-1(mod q) ->pelo pequeno teorema de Fermat ->  (2^(P*3^k))^q =
-1 = 2^(P*3^k) (mod q)
Logo

 2^(2*P*3^k) = 1 (mod q)

Seja r uma raiz primitiva mod q e seja u tal que r^u=2 (mod q), com 0q=1(absurdo) ou q=3(absurdo por definicao de q)

Portanto não existe um tal n.
---

Bruno Leite






O quarto problema da IMO 2000

2000-08-09 Por tôpico Bruno Leite

Oi pessoal

Eu ACHO que consegui fazer o problema quatro da IMO 2000 e estou enviando-o
para a lista para saber se esta tudo certinho.

Eu vou precisar de um lemazinho:

Se A esta contido em {1,2,3,...,100} e B esta contido em {1,2,3,...,100},
entao 

#{x+y | x pertence a A e y pertence a B}>=#A+ #B -1

Prova: Seja i=#A e j=#B. Eh so'  ordenar os elementos de B e fazer inducao
sobre j.

--

Agora, sejam A , B e  C conjuntos disjuntos 2 a 2 tais que A U B U
C={1,2,3,...,100}. Sejam #A=a, #B=b e #C=c. Eh claro que a+b+c=100.
Dados 2 conjuntos P e Q vou definir S(PQ) como sendo #{x+y | x pertence a P
e y pertence a Q}

Entao S(AB)+S(BC)+S(AC)>=a+b-1+b+c-1+a+c-1=197.
Sabemos que os possíveis valores da soma dos cartoes escolhidos pelo membro
da platéia estao entre 3 e 199 (ou seja, sao 197 valores)
Portanto todos os valores de {3,4,5,...199} devem ser "atingidos" pelas
somas, ou haverá somas idênticas entre pares diferentes de conjuntos, e aí
o mágico nao pode adivinhar nada. Como em particular 3 deve ser "atingido",
1 e 2 devem estar em conjuntos diferentes. Analogamente, 4, 199 e 198 devem
ser atingidos e por isso 1 e 3 pertencem a conjuntos diferentes, 100 e 99
também, 100 e 98 também.

Podemos supor por simetria que 1 pertence a A.
Caso 1: 2 pertence a B e 3 pertence a C

Eh facil ver que isso gera a seguinte particao:
A={1,4,7,10,...,100}
B={2,5,8,11,...,98}
C={3,6,9,12,...,99}
que eh uma solucao do problema.(Basta provar que 4 TEM que pertencer a A, 5
TEM que pertencer a B, etc, e generalizar)

Caso 2: 2 pertence a B e 3 pertence a B.
Dividimos em tres sub-casos:
a)100 pertence a C

Temos entao obrigatoriamente 99 e 98 pertencentes a B
Agora, 97 nao pode pertencer a C ou a soma 197 nao poderá mais ser
atingida. Mas se 97 pertence a A, entao temos que 96 pertence a A, 95
pertence a A, etc... 4 pertence a A. Mas isso eh absurdo pois teremos duas
somas iguais envolvendo conjuntos diferentes: 100+3=99+4. Por isso 97
pertence a B.

Aplicando varias vezes essa ideia chegaremos à seguinte partição:
A={1}
B={2,3,4,...98,99}
C={100}
que eh a outra solucao do problema

b)100 pertence a A
Isso leva a um absurdo:
Seja x o maior elemento de C. É facil eliminar as possibilidades de x+1
pertencer a  A ou a B. Logo x+1 pertence a C, mas isso contradiz o fato de
x ser o maior elemento de C.

c)100 pertence a B
Usando o mesmo raciocinio (supor x=max(C) etc) chegamos a outro absurdo.


Portanto as unicas solucoes sao 

A={1,4,7,10,...,100}
B={2,5,8,11,...,98}
C={3,6,9,12,...,99}

e

A={1}
B={2,3,4,...98,99}
C={100}

salvo as simetrias entre A, B e C.


Bruno Leite




Re: regulamento-OBF

2000-08-04 Por tôpico Bruno Leite

At 20:59 04/08/00 -0300, you wrote:
>Por favor, apesar desta página ser sobre  matemática,
>gostaria de saber apenas  onde posso encontrar mais
>informações sobre a OBF. Se  possível, em algum
>website.   Muito obrigado!   Bruno Mintz-Mensagem original-

Veja http://www.sbf1.if.usp.br/olimpiadas/obf2000/

>De:  <[EMAIL PROTECTED]>
>Para:  discussã<[EMAIL PROTECTED]>
>Data:  Sexta-feira, 4 de Agosto de 2000 04:47
>Assunto:  regulamento-OBF
>
> Um colega leu o regulamento da  Olimpíada Brasileira de
>Física e achou  interessante: - na primeira fase: de 30
>questões o  candidado escolhe 20 - na segunda fase: das 16
>questões o  candidato escolhe 8 O que vocês acham
>desse tipo de prova na OBM com  algumas adaptações se for
>o caso? Segundo ele é um estímulo a mais para o
>candidato, fora  a tranquilidade, por poder escolher as  questões.

Na verdade esse regulamento só vale para o 1º e 2º colegial, porque em
certas escolas ensina-se mecânica no 1º e óptica e ondas no 2º e em outras
é o contrário, por isso não seria possivel fazer uma olimpíada para o
primeiro colegial por exemplo.

No 3º colegial você tem que fazer todas as questões mesmo.

A OBM desse ano estava tão difícil que o que eu fiz foi escolher uma
questão ou outra e deixar o resto em branco :o)



Bruno Leite





IMO 2000

2000-07-19 Por tôpico Bruno Leite

As provas da IMO-2000 já estão disponíveis em algum lugar?




Re: uma desigualdade!

2000-07-13 Por tôpico Bruno Leite

...
>
>3) Como achar o limite superior 1.202057 ?


Você pode ver http://www.lacim.uqam.ca/piDATA/Zeta3.txt

Eu não sei como achar esse limite com papel e caneta.
Quero dizer, não sei se realmente temos que fazer muitas contas ou se
alguma boa idéia nos leva rapidamente ao resultado.

>[]s
>Luís Lopes
>
>
>
>




Re: uma desigualdade!

2000-07-13 Por tôpico Bruno Leite

At 15:30 13/07/00 -0300, you wrote:
>Saudações a todos,
>
>Para que saibamos do que vou falar, copio a mensagem recebida:
>
>
>> On Mon, 10 Jul 2000 17:02:23 -0300
>> Bruno Leite <[EMAIL PROTECTED]> wrote:
>> >At 22:07 09/07/00 -0300, you wrote:
>> >>Caros amigos, como posso verificar a desigualdade
>> >>  1/1^3  + 1/2^3  + 1/3^3  + ...+ 1/n^3 <3/2   para todo
>> >n natural ?
>> >
>> >Um esbo=E7o de solu=E7=E3o:
>> >Provar por indu=E7=E3o que  1/1^3  + 1/2^3  + 1/3^3  + ...+
>> >1/n^3 <3/2(1-1/n)
>> >para n>1
>> >
>> >Ent=E3o quando n->infinito, 1/1^3  + 1/2^3  + 1/3^3  + ...+
>> >1/n^3<3/2
>> >
>> >A s=E9rie 1/1^3  + 1/2^3  + 1/3^3  + ...+ 1/n^3 =E9 crescente,
>> >limitada
>> >superiormente e tem um limite que =E9 menor que 3/2.
>> >Logo para qualquer n natural 1/1^3  + 1/2^3  + 1/3^3  +
>> >...+ 1/n^3 <3/2.
>> >
>> >Na verdade vale 1/1^3  + 1/2^3  + 1/3^3  + ...+ 1/n^3
>> ><1.202057
>> >
>> >Abra=E7o
>> >
>> >Bruno Leite
>
>
>1) No livro The Art of Computer Programming Vol 1, de D. Knuth,
>temos o seguinte resultado:
>
>Um limite superior para a soma S = 1/i^r, i=1,2,...n com r>1 e real é dado
>por 2^{r-1}/(2^{r-1}-1). Colocando r=3, obtemos S < 4/3<3/2.

Essa desigualdade é muito boa! Você tem uma demonstração?

>2) Gostaria de ter mais detalhes para a prova por indução.

Suponha que 1/1^3  + 1/2^3  + 1/3^3  + ...+ 1/n^3 <3/2(1-1/n).
Então 1/1^3  + 1/2^3  + 1/3^3  + ...+ 1/n^3
+1/(n+1)^3<3/2(1-1/n)+1/(n+1)^3=3/2[(1-1/n)+2/(3*(n+1)^3)]<(*)3/2(1-1/(n+1))

(*)Temos que provar que -1/n+2/(3*(n+1)^3)<-1/(n+1)
Multiplicando por n+1

-1/n+2/(3*(n+1)^3)<-1/(n+1) <-->
-1-1/n+2/(3*(n+1)^2)<-1  <-->  2/(3*(n+1)^2)<1/n  <-->  2n<3n^2+6n+3  <-->
3n^2+4n+3>0 <-->delta<0, o que é verdade.

>3) Como achar o limite superior 1.202057 ?
Você pode usar o Maple ou fazer um programinha em C

"for(i=1;i<100;i++)
soma=soma+1/(i*i*i);"

por exemplo.

Talvez haja algo de bom em
http://www.mathsoft.com/asolve/constant/apery/apery.html

>[]s
>Luís Lopes
>
>
>
>




Re: uma desigualdade!

2000-07-10 Por tôpico Bruno Leite

At 22:07 09/07/00 -0300, you wrote:
>Caros amigos, como posso verificar a desigualdade
>  1/1^3  + 1/2^3  + 1/3^3  + ...+ 1/n^3 <3/2   para todo n natural ?

Um esboço de solução:
Provar por indução que  1/1^3  + 1/2^3  + 1/3^3  + ...+ 1/n^3 <3/2(1-1/n)
para n>1

Então quando n->infinito, 1/1^3  + 1/2^3  + 1/3^3  + ...+ 1/n^3<3/2

A série 1/1^3  + 1/2^3  + 1/3^3  + ...+ 1/n^3 é crescente, limitada
superiormente e tem um limite que é menor que 3/2.
Logo para qualquer n natural 1/1^3  + 1/2^3  + 1/3^3  + ...+ 1/n^3 <3/2.

Na verdade vale 1/1^3  + 1/2^3  + 1/3^3  + ...+ 1/n^3 <1.202057


Abraço

Bruno Leite







Re: apreciaçãofjbw==

2000-07-05 Por tôpico Bruno Leite

(...)
4. Se a for ímpar e b for ímpar (absurdo: ímpar não divide par)

Isso não é verdade**

   (...)
-

Suponha que mdc(a,b)=g, e que a=gA, b=gB. 
é claro que mdc(A,B)=1.

Se (a^2+b^2)/ab é inteiro, então (A^2+B^2)/AB é inteiro.
Em particular, (A^2+B^2)/A é inteiro e por isso A divide B^2, mas A é primo
com B e portanto A=1.
Analogamente B=1 e por isso a=b.(está tudo certo, não?)

Bruno Leite






Dica

2000-06-18 Por tôpico Bruno Leite

Oi pessoal

Queria uma dicazinha de como eu posso provar que "existem infinitos n tal
que 2n^2+2n+1 eh quadrado perfeito"

Obrigado

Bruno Leite




Re: Desenho mais Geometria

2000-02-16 Por tôpico Bruno Leite

At 17:53 16/02/00 -0400, you wrote:
>Como resolver?
>Dados dois pontos A e B em semiplanos distintos em relacao a uma reta r,
>encontre um ponto M sobre r tal que
>a diferenca entre as distancias MA e MB seja minima (em módulo).
>
É só traçar a mediatriz de AB e ver onde ela encontra a reta r. Esse é o
ponto M desejado pois então teremos MA-MB=0

Na verdade é preciso considerar o caso de a mediatriz e r serem paralelas,
mas isso não deve ser muito difícil!!!

Bruno Leite
>
>
>
>
>
>
>
>
>
>
>
>
>
>



Re: como provar (polígonos)?

2000-02-08 Por tôpico Bruno Leite

At 22:53 07/02/00 -0200, you wrote:
>que as alturas de um triângulo se cortam em um ponto? Isto vale para
>demais polígonos (acho que só convexos). Há outra limitação?
>Ah, aproveitem e demonstrem que as bissetrizes e medianas tb se
>intersectam.
>
>Abraço,
>
>Benjamin Hinrichs

É MUITO fácil provar que as medianas concorrem num único ponto, se vc usar
o Teorema de Ceva.

Bissetrizes:

b(A) é a bissetriz de A

b(A) equidista de AB e AC, e b(B) equidista de BA e BC. Seja J o ponto em
que elas se cortam. J equidista de AB, AC, BC, em especial de CA e CB.
Portanto J pertence à terceira bissetriz!

Para as alturas use também o teorema de Ceva lembrando que o pé da altura
divide o lado AB em dois pedaços de tamanho b.cosA   ea.cos B.

A propósito o Teorema de Ceva é assim:

Seja P um ponto interno do triângulo. AP corta BC em A`, BP corta AC em B`,
CP corta AB em C`.

Então BA`   xCB`   xAC` = A`C   xB`A   x   C`B

Por outro lado, se essa relação vale então AA`, BB`e CC` se cortam em P.


Bruno Leite



pi(x) e primos da forma 4k+1

2000-02-06 Por tôpico Bruno Leite

>Outro Problema, Alguém conhece a  função pi(x)? Quer me explicar?
>Tem um exercício, porém não entendi direito.
>
>O exercício pede para Provar que há infinitos número primos congruentes a 1
>mod 4.
>
>
>
>Muito Obrigado!
>
>Marcos Eike Tinen dos Santos
>
Caro Marcos,

A função pi(x) que eu conheço (e torço para que estejamos falando da mesma
função!)
é definida assim:

pi(x)= número de primos menores que x. (ou será que é menor OU IGUAL? -
alguém da lista pode confirmar))

Uma prova de que existem infinitos primos da forma 4k+1 é a seguinte:

Suponha que existam n primos dessa forma:p1, p2, ..., pn.

Considere o número X=(p1.p2. ... .pn)^2 + 1. Ele é maior que qualquer primo
da forma 4k+1 e portanto por hipótese é composto.

Notando que ele não é uma potência de 2(pois é congruente a 2 no módulo 4)
então algum primo ímpar q o divide: X=0(mod q)

Suponha que q é da forma 4k+3. Então X=0(mod q) -> (p1.p2. ... .pn)^2 = -1
(mod q)

Elevando tudo a 0,5(p-1), temos (p1.p2. ... .pn)^(p-1) = -1^(0,5(p-1)) (mod
q) 

Como p=4k+3 por hipótese,

(p1.p2. ... .pn)^(p-1) = -1^(0,5(4k+2)) (mod q)

(p1.p2. ... .pn)^(p-1) = -1^(2k+1) (mod q)

No entanto o lado esquerdo dessa igualdade vale 1 (pelo pequeno teorema de
Fermat)
enquanto o direito vale -1. Isso é,
1=-1(mod q) o que implica em q=2 ou q=1, um absurdo pois q é primo ímpar.

Assim, q é da forma 4k+1 e portanto q pertence ao conjumto (supostamente
finito) {p1, p2, ..., pn.}. Assim, q=px (x é um índice de p, não está
multiplicando p), com 1<=x<=n

Temos X=0(mod q) -> (p1.p2. ... .pn)^2 + 1=0(mod px) -> 1=0(mod px) ABSURDO.

Portanto existem infinitos primos da forma 4k+1!!!

Bruno Leite



Re: Entendendo à Aplicação dos números complexos

2000-02-06 Por tôpico Bruno Leite

At 00:13 04/02/00 -0200, you wrote:
>
>Oi,
>
>Quem poderia me explicar o uso desta teoria? Li o artigo no Eureka, entendi
>a parte que eles mostram as raízes complexas dispostas na circunferência
>tendo n lados. Porém, não entendi à aplicação nos exercícios.
>Os números complexos se não estou enganado pode ser usado como vetores,
>aguém poderia me explicar?
>Caso seja possível por favor uma explicação completa. Será que encontro aqui
>no Brasil totalmente ligado ao assunto?
>
>Outro Problema, Alguém conhece a  função pi(x)? Quer me explicar?
>Tem um exercício, porém não entendi direito.
>
>O exercício pede para Provar que há infinitos número primos congruentes a 1
>mod 4.
>
>
>
>Muito Obrigado!
>
>Marcos Eike Tinen dos Santos

Caro Marcos e Lista

Eu também não entendi nada do artigo dos números complexos! Na escola
aprendemos que os números complexos servem para resolver equações de 2º
grau com delta<0 e também vemos o plano de Gauss (eixo real e imaginário, etc)

É claro que nenhum bom aluno deve limitar-se ao que aprende nessas aulinhas
de colegial, mas admitir que os alunos "olímpicos" já sabem as aplicações
vetoriais dos complexos talvez seja uma visão muito pretensiosa! (sem
querer ofender, é claro...)

Eu já estudei vetores, produto vetorial, produto escalar, vetores na geom.
analítica, etc, mas não pude entender o artigo justamente pela falta de uma
introdução em que se poderia ter definido o que é um vetor na "linguagem"
dos números complexos, e mostrado explicitamente como se muliplicam esse
números-vetores (são as mesmas regras dadas na escola?)

Sem essa introdução o "campo de ação" do artigo fica muito mais limitado:
apenas três ou quatro estudantes entenderão perfeitamnete o que foi
explicado! Peço à Comissão de Olimpíadas que prepare um texto que "conecte"
a matéria da escola com o conteúdo desse artigo. Isto é, um texto que
explique a teoria sobre números complexos desde o último assunto que as
escolas normalmente dão (representação trigonométrica, soluções de
x^n=1,operações na forma trigonométrica, resolução de qualquer equação
quadrática) até o que é requerido pelo autor. Eu acho que não deve ser
muita coisa, se um ponto ou outro forem esclarecidos a grande maioria dos
alunos já poderá se virar.

Agradeço a quem der uma ajuda explicando o artigo.

Abração a todos da lista

Bruno Leite

PS-Sobre pi(x) e o exercício, eu já mando um email.



Re: Quem a maior???

2000-02-06 Por tôpico Bruno Leite

At 10:00 06/02/00 -0200, you wrote:
>Marcelo Souza escreveu:
>> 
>> Olá pessoal,
>> 
>>Este problema aki eu naum tenho a mínima idéia de como
resolvê-lo. se
>> alguem puder ajudar:
>> Quem é maior 1000^1000 ou 1001^999?
>> obrigado
>> abraços
>> marcelo
>> 
>> __
>> Get Your Private, Free Email at http://www.hotmail.com
>Vamos dividi-los para ver se dá maior que 1.
>1001^999 / 1000^1000 = [(1001/1000)^1000]/1001 = [(1+0,001)^1000]/1001.
>O numerador da fração é o milésimo termo da famosa sequencia crescente
>cujo limite é o número e. Portanto a fração é menor que e/1001 e é muito
>menor que 1.
>
>Logo 1001^999 é o maior.

Mas, se a fração é menor que 1, então o denominador 1000^1000 é maior!!

Bruno F.C. Leite



Re: Báskara

2000-01-12 Por tôpico Bruno Leite

At 01:06 12/01/00 -0200, you wrote:
>Eu queria que me fizessem um favor. Eu nunca vi a demonstracao da formula de
>Baskara para a resolucao de equacoes do 2º grau. Voces poderiam me
>mostra-la?
>
>David
>
Eu conheço assim:

ax^2+bx= -c

4a^x^2+4abx=-4ac

4a^x^2+4abx+b^2=b^2-4ac

(2ax+b)^2=sqrt(b^2-4ac)

etc

x=-b+-sqrt(b^2-4ac)/2a

A idéia foi formar trinômio quadrado perfeito do lado esquerdo da
igualdade. Por isso passa-se c para o outro lado, multiplica-se por 4a,
soma-se b^2 e extrai-se a raiz.


Bruno Leite



Re: trapezios - corrigindo

2000-01-12 Por tôpico Bruno Leite

At 08:48 11/01/00 -0200, you wrote:
>At 03:36 09/01/00 -0200, you wrote:
>>
>>  Pessoal, ha um teorema q eu nao sei direito, que diz algo assim: a soma
>>dos quadrados das diagonais de qualquer trapezios eh igual a soma dos
>>quadrados dos lados nao paralelos mais o produto dos lados paralelos... nao
>>tenho certeza, alguma coisa assim... alguem pode me ajudar dizendo como eh
>>corretamente esse teorema e qual seu nome? e onde posso encontrar sua
>>demonstraçao?
>>
>>agradeço desde já
>>
>>
>>
>>Maurício Paiva
>>Belém-PA
>>[EMAIL PROTECTED]
>>
>
>Tem um Teorema (eu acho que se chama Teorema de Ptolomeu) que diz "em
>qualquer quadrilátero inscritível de vértices A, B, C, D (nessa ordem
>cíclica) com AB=m, BC=n, CD=p, DA=q, AC=r e BD=s (AC e BD são diagonais),
>vale a relacão mp+nq=rs. Se o quadrilátero não é inscritível sempre teremos
>mp+nq>rs.
>
>Como todo trapézio é inscritível, vale o Teorema de Ptolomeu. Há uma
>demonstração na página 24 da Eureka!5
>
>Bruno Leite
>
Falei besteira. Na verdade, eu imaginei um trapézio isósceles quando
escrevi isso. O certo é(obrigado, prof.Barone)
"Como todo trapézio isóceles é inscritível, pois a soma dos ângulos opostos
é 180, vale o Teorema de Ptolomeu"



Re: trapezios

2000-01-11 Por tôpico Bruno Leite

At 03:36 09/01/00 -0200, you wrote:
>
>   Pessoal, ha um teorema q eu nao sei direito, que diz algo assim: a soma
>dos quadrados das diagonais de qualquer trapezios eh igual a soma dos
>quadrados dos lados nao paralelos mais o produto dos lados paralelos... nao
>tenho certeza, alguma coisa assim... alguem pode me ajudar dizendo como eh
>corretamente esse teorema e qual seu nome? e onde posso encontrar sua
>demonstraçao?
>
>agradeço desde já
>
>
>
>Maurício Paiva
>Belém-PA
>[EMAIL PROTECTED]
>

Tem um Teorema (eu acho que se chama Teorema de Ptolomeu) que diz "em
qualquer quadrilátero inscritível de vértices A, B, C, D (nessa ordem
cíclica) com AB=m, BC=n, CD=p, DA=q, AC=r e BD=s (AC e BD são diagonais),
vale a relacão mp+nq=rs. Se o quadrilátero não é inscritível sempre teremos
mp+nq>rs.

Como todo trapézio é inscritível, vale o Teorema de Ptolomeu. Há uma
demonstração na página 24 da Eureka!5

Bruno Leite



Re: é o chato novamente

2000-01-02 Por tôpico Bruno Leite

Caro Benjamin,

Vou usar == como o sinal de "é congruente a".
Se a==b (vamos deixar em todas as congruências um "mod x" subentendido) e
c==d, então 
(1)a+c==b+d
(2)ac==bd
(3)a^n==b^n
Se usarmos (2) repetidamente achamos (3):( aa==bb, a^3==b^3,... --> a^n==b^n )


>Isto quer também dizer que 3 = -1 mod 4 (onde = querem dizer três
>tracinhos ao invés de dois), ou seja a = -1 mod (a+1)?

Falando em mod 4, todos os termos da PA (...,-9,-5,-1,3,7,11,...) são
congruentes entre si.  -1 é um termo "nobre" pois é fácil calcular sua
potências.

>Eu me pus a provar hoje de manhã que a^n -(-1)^n mod a+1 = 0 (o que já
>faz mais minha linguagem do que congruências), mas fui infeliz em minha
>tentativa. 

É só colocar (mod a+1) a==-1 --> a^n==(-1)^n --> a^n - (-1)^n=0 [as
propriedades (1), (2) e (3) permitem tratar o sinal de congruência como se
fosse um de igual, com poucas exceções (exemplo: não se pode dividir os
dois membros da equação por um mesmo número, em geral)]

As dem. de (1), (2) e (3) podem ser achadas na internet, na Eureka2 ou (se
não me engano) no artigo de teoria dos números do Nicolau Saldanha, que
está lá na HP dele. Aliás, nessas fontes vc pode achar um monte de outras
coisas legais...

Um problema legal em que se pode usar congruências é (IMO 75)"seja
x=^. Sejam A=soma dos algarismos de x; B=soma dos algarismos de A.
Qual é a soma dos algarismos de B?"

Bruno Leite







A Razão Áurea

1999-12-21 Por tôpico Bruno Leite

>From: [EMAIL PROTECTED]
>Reply-To: [EMAIL PROTECTED]
>To: [EMAIL PROTECTED]
>Subject: Re. Ainda sobre sen 36
>Date: Tue, 21 Dec 1999 23:27:44 -300
>
>Para encontrar o seno e o cosseno de 36 eu utilizo o triângulo áureo 
> >(isósceles com ângulo do vértice 36 graus)ABC. Traço a bissetriz de >um 
>dos ângulos da base (BD) e tenho uma semelhança entre os >triângulos BDC e 
>ABC. Se a base BC mede x e os outros lados medem 1, >temos: (1-x)/x  = x . 
>portanto x =[sqrt(5) - 1]/2 então traçando a >altura encontro sen 18 = 
>[sqrt(5) - 1]/4
>encontrar o cosseno de 36 deve ser  fácil.
>
>O número [sqrt(5) - 1]/2  é chamado PHI assim como phi = >[1-sqrt(5)]/2 
>esses
>
>números (PHI e phi) têm várias prorpiedades que já devem ter sido 
> >discutidas nessa lista (se não deveria) e são chamados números de >ouro!!
>Por exemplo a razão entre números consecutivos da sequancia de >fibonacci é 
>PHI!
>
>
>PHI = 1+1/(1+1/(1+1/...)
>
>Espero ter ajudado!
>
>Marcos Paulo
>http://unimail.unisys.com.br

O número PHI, como disse o Marcos, ou "golden mean", na verdade é o LIMITE 
(n tendendo a infinito)de f(n+1)/f(n) sendo f(n) e f(n+1) termos 
consecutivos da sequência de Fibonacci (0,1,1,2,3,5,8,13,21,34,55,etc)

Outras propriedades interessantes desse número podem ser achadas em
http://www.mathsoft.com/asolve/constant/gold/gold.html

Uma delas é:
se  g(1)=1
e   g(n+1)=sqrt[g(n)+1]

Temos, para n=infinito, g(n)=a razão áurea (ou PHI, ou golden number)

É claro que esse número é 
sqrt(1+sqrt(1+sqrt(1+sqrt(1+sqrt(1+sqrt(1+sqrt(1+sqrt(1+sqrt(1+...

Para terminar, esse número aparece bastante na arquitetura "clássica". Eu já 
vi uma vez um livro que destacava partes de várias construções da 
antiguidade (teatros, monumentos, edifícios em geral, etc) onde essa razão 
aparecia frequentemente.

Até dia 2 de janeiro.(estarei ausente 10 ou 11 dias)
Feliz Natal a todos!

Bruno Leite
__
Get Your Private, Free Email at http://www.hotmail.com



As matérias do 1º ano de Matemática

1999-12-20 Por tôpico Bruno Leite

Eu queria saber o que é ensinado no 1º de Faculdade de Matemática, curso 
Bacharelado. Eu tento consultar a página do Inst. de Mat. e Estat. da USP 
www.ime.usp.br mas é uma bagunça. Algum dos mestres da lista poderia me 
enviar uma lista mais ou menos detalhada do que é estudado no 1º ano?

Agradeço,

Bruno Leite
__
Get Your Private, Free Email at http://www.hotmail.com



Re: taxa de importação

1999-12-18 Por tôpico Bruno Leite

>Bruno Leite Wrote:
>
> >Não há exemplares desse livro na USP nem na UNICAMP. Na Amazon.com > >tem 
>o livro, mas é meio caro(33 dólares x 1,90 + taxa de importação > >60% etc) 
>e
> >deve demorar a chegar, se encomendado.
>
> Sei q não tem nada a ver com matemática, mas como não tenho a >quem 
>perguntar, vai aqui mesmo..
>Tenho ouvido falar dessa tal taxa de importação, e não faço idéia de >como 
>ela se aplica. Já vi gente reclamando q pagou isso em cd's,

Eu já comprei CDs lá e tive que pagar 60% sobre o preço dos produtos.

>outros em livros, mas eu pedi um livro na Amazon a menos de um mês >atrás 
>(24 dólares com a taxa de remessa) e não me cobraram nada de >taxa de 
>importação!
> Será que alguém pode me explicar isso ou indicar um lugar onde >se 
>possa obter uma informação bem clara a respeito??

Acho que em www.fazenda.gov.br. Mas parece que é isso aí que falaram: livros 
não pagam(eu não sabia que havia isenção) mas CDs pagam (muito)

Bruno Leite
__
Get Your Private, Free Email at http://www.hotmail.com



Re: Números Complexos, Geometria e Edmilson

1999-12-15 Por tôpico Bruno Leite

At 19:24 15/12/99 -0200, you wrote:
>Tem um livro, mas não sei onde conseguir aqui no Brasil.
>No final do artigo tem uma indicação! OK?
>Caso consiga Aqui, no Brasil, me dá um toque.
>

Não há exemplares desse livro na USP nem na UNICAMP. Na Amazon.com tem o
livro, mas é meio caro(33 dólares x 1,90 + taxa de importação 60% etc) e
deve demorar a chegar, se encomendado. O link é 
http://www.amazon.com/exec/obidos/ASIN/0883855100/qid=945310706/sr=1-2/002-8
962006-1271417.

>Vc entendeu como ele deduziu aquela fórmula w^2+w+1=0,
>para explicar a aplicação num triângulo equilátero?

Eu ***acho*** que é porque as raízes da equação (x^3 - 1)=0  --> (x-1)(x^2
+ x + 1)=0 formam aquele triângulo, no plano complexo.

Bruno Leite



Álgebra IV (ou já é o V ?) e outras coisas

1999-12-15 Por tôpico Bruno Leite

Trata-se de um problema do Segundo Teste de Seleção da IMO-Ibero.
Para reais positivos satisfazendo a+b+c=abc, mostre que 

(a^2 + 1)^(-0,5) + (b^2 + 1)^(-0,5) + (c^2 + 1)^(-0,5) <= 1,5

Determine quendo ocorre a igualdade.


Prova do ITA de matemática
A prova do ITA foi muito feia e cheia de contas. Por exemplo, "Um cone
circular reto com altura sqrt8 cm e raio da base 2 cm está inscrito numa
esfera que por sua vez está inscrita num cilindro. A razão entre as áreas
das superfícies totais do cilindro e do cone é? (era um teste)" É só conta!

O que eu queria saber é se tem um jeito bom, razoável, de fazer a questão 24.
Eu acho que era a mais difícil, e eu desisiti dela porque realmente cheguei
a uma quantidade absurda de contas e equações. Eu acho que a prova está
disponível na Internet, mas se não for o caso eu mando a questão.


Eu achei esse texto em alguma página da Internet. Eu achei isso muito
surpreendente, muito mesmo. Alguém sabe demonstrar isso? (eu não me lembro
de onde eu peguei isso)


"K. Brown describes a fascinating number-theoretic function f(n). Take any
positive integer n, round it up to the nearest multiple of n-1, then round
this result up to the nearest multiple of n-2, then (more generally) round
the kth result up to the nearest multiple of n-k-1. Stop when k=n-1 and let
f(n) be the final value. For example, f(10)=34 since 

  10-->18-->24-->28-->30-->30-->32-->33-->34-->34. 

  The ratio n^2/f(n) approaches pi as n increases.[!!]


Bruno Leite







Re: Números Complexos, Geometria e Edmilson

1999-12-15 Por tôpico Bruno Leite

>Por favor necessito de explicação dos números complexos à Geometria.
>
>Muito Obrigado

Eu também necessito! Eu li o artigo do Edmilson Motta na Eureka6 e não
entendi nada! Alguém tem uma indicação de um livro que dê alguma base para
entender aquilo? Será que um livro de vetores adianta? 

Bruno Leite




Sobre campeonatos de futebol em Tumbólia

1999-12-14 Por tôpico Bruno Leite

É a questão 5 da OBM-1999(3ºnível)

Na página da OBM está o seguinte enunciado(era esse enunciado mesmo?eu
estou achando que há alguma pequena diferença)

Em Tumbólia existem n times de futebol .
Deseja-se organizar um campeonato em que cada time joga exatamente uma vez
com cada um dos outros. Todos os jogos ocorrem aos domingos, e um time não
pode jogar mais de uma vez no mesmo dia. Determine o menor inteiro positivo
m para o qual é possível realizar um tal campeonato em m domingos.


De qualquer modo, eu resolvi esse problema de um jeito, um amigo meu fez de
outro jeito, mas depois ele veio falar para mim que as duas soluções
estavam erradas. Eu ainda não consegui ver o que está errado na minha
solução, e não deu para falar com esse meu amigo muito direito. Eu não vou
mandar a minha solução para a lista, mas eu queria ver como é que vocês
fizeram. E eu que pensei que foi a questão mais fácil...(ou menos difícil?)

Bruno Leite
(Agora com novo email: [EMAIL PROTECTED])


PS Por que esta lista está parada? Acho que não recebi sequer 10 mensagens
nesses últimos 15 dias...Há algum problema técnico?



Problema de função

1999-12-12 Por tôpico Bruno Leite

Aí vai:Mostre que não existem funções f:R->R tais que f(f(x))=x^2-1996

Bruno Leite

__
Get Your Private, Free Email at http://www.hotmail.com



Re: número algébrico

1999-12-08 Por tôpico Bruno Leite


>Encontrar sen e cos de 1 a partir de sen e cos de 3 não seria equivalente a
>trissectar o ângulo?
>isso é impossível!

Vou falar só do seno; o cosseno merece as mesmas considerações.
É possível chegar à equação do terceiro grau:(é o arco triplo)
sen3= 3 sen1 - 4(sen1)^3  onde a incógnita é o sen1. Como eu disse, o sen3 é 
um termo independente perfeitamente "achável" mas vamos deixá-lo nesta forma 
pois eu o calculei ano passado e me lembro que ele tinha muitos radicais...

Manipulando a equação (elevando ao quadrado até todos os radicais do sen3 
sumirem, passando termos de um lado para o outro, etc) nós provamos que a 
equação em sen1 pode ser colocada como equação polinomial de coeficientes 
inteiros, e portanto sen1 é algébrico.
Chamo a atenção para o seguinte: NÓS SÓ MONTAMOS A EQUAÇÃO; NINGUÉM VAI 
RESOLVÊ-LA: se vc tentar a fórmula de Cardano, verá que não dá em nada. O 
número sen1 é algébrico, mas não dá para colocá-lo na forma de raízes 
quadradas, cúbicas, etc. Um livro que explica tudo isso melhor é o já 
citado(em outros emails da lista) "Romance das equações algébricas". Lá ele 
dá exemplos do que acontece se vc tentar resolver uma equação como esta, 
tentando prosseguir usando "artimanhas" algébricas : vc começa a "andar em 
círculos".

Uma coisa interessante é que dá sempre para achar sen 3k, cos 3k, para 
qualquer k inteiro. É curioso mas dá para calcular tan 129(!), por exemplo.
É claro que se o número não for múltiplo de três não dá para 
calculá-lo.(algebricamente, eu quero dizer) Se fosse possível calculá-lo 
acharíamos sen1 e cos1.

Bruno Leite

*Eu acho que não escrevi nenhuma besteira, mas se alguém perceber alguma, me 
avise por favor.

__
Get Your Private, Free Email at http://www.hotmail.com



Re: número algébrico

1999-12-08 Por tôpico Bruno Leite


>   Como faço para mostrar que o seno de um grau é um número
>algébrico?

É possível achar sen36 e cos36***.  Como vc sabe sen30 e cos30, pelas 
fórmulas sen(36-30)=sen36 cos30 - sen30 cos36 e cos(36-30)=cos36 cos30 + 
sen30 sen36   vc pode achar sen6 e cos6.

Usando arco metade, pode-se achar sen3 e cos3 (são radicais enormes)

Agora, use o arco triplo para forçar uma equação em sen1 ou cos1. É claro, 
será de 3º grau.

Eu não tive a coragem de fazer as contas, mas depois de tudo isso, se vc 
manipular a equação, vai cair numa equação em sen1 ou cos1, com coeficientes 
inteiros.(o que mostra que sen1 é algébrico)

***sobre sen36 e cos36

Você pode achar cos36 desenhando um pentágono regular e suas diagonais (a 
estrela de 5 pontas). Há um montão de triângulos semelhantes e outro tanto 
de triângulos isósceles na figura, dá para achar a relação entre a diagonal 
e o lado.
Agora, examine o triângulo retângulo de vértices (1),(2) e (3), onde
(1) é vértice do pentágono
(2) é ponto médio de um lado adjacente ao vértice de (1)
(3) é vértice do pentágono interno, o mais próximo de (1) e (2).
Nesse triângulo retângulo, há um ângulo de 36, e os lados são conhecidos. 
assim vc pode calcular cos36.


Bruno Leite

__
Get Your Private, Free Email at http://www.hotmail.com



Algebra III

1999-12-01 Por tôpico Bruno Leite

Oi, pessoal da lista,

Arrumei dois probleminhas legais, de álgebra. O primeiro eu fiz, a resposta 
está no fim deste e-mail. Não tive nenhuma boa idéia para o segundo, ele 
está impossível de resolver. Quem conseguir resolvê-lo pode mandar um email 
que eu ficarei muito grato.

1)Determine condições nos inteiros b e c para que os três polinômios x^2 + 
bx + c; x^2 + bx + (c + 1); x^2 + (b + 1)x + c sejam fatoráveis nos 
inteiros.

2)Resolva nos reais.

sqrt(1 - x) = 2.x^2 - 1 + 2x.sqrt(1 - x^2)  [o ponto é sinal de vezes]

(muito) boa sorte

Bruno Leite


***A minha resposta para o 1) está logo abaixo, parece que está certa.


b=2(n^2 + n - 1) e  c=(n^2 + n - 1)^2 - 1, para qualquer n

__
Get Your Private, Free Email at http://www.hotmail.com



Para fazer no Domingo

1999-11-27 Por tôpico Bruno Leite

Estou mandando ainda mais dois problemas, e como sempre, não sei 
resolvê-los.

Sejam d1,d2,...,dk (1,2,...,k são índices de d) todos os divisores positivos 
de n natural. onde 1=d1=4 e d1^2+d2^2+d3^2+d4^2=n.

Prove que existe um inteiro positivo k tal que k(2^n) + 1 é composto para 
qualquer n(inteiro positivo).

Boa sorte!

Bruno Leite

__
Get Your Private, Free Email at http://www.hotmail.com



Meio problema

1999-11-27 Por tôpico Bruno Leite

(a) Para quais inteiros positivos n existe um conjunto Sn de n inteiros 
positivos distintos tal que a média geométrica de qualquer subconjunto de Sn 
é um inteiro?

Esse é fácil: para todo n. Basta escolher, por exemplo, {1^n! , 2^n!, 3^n!, 
..., n^n!}- com o fatorial no expoente. É claro que esse conjunto serve. 
Agora vejam o que se pede depois.

(b)Existe um conjunto infinito S de inteiros positivos tal que a Med.Geom. 
de qualquer subconjunto FINITO de S é inteiro?

Esse eu não sei fazer, mas desconfio (fortemente) que não existe. O duro é 
provar. Alguém se dispõe?

Bruno Leite

__
Get Your Private, Free Email at http://www.hotmail.com



Álgebra II

1999-11-25 Por tôpico Bruno Leite

De novo, peço ajuda pois estes aqui são piores que os outros!

1)fatore:
a)x+1, para x>0 (???)
b)x^4 + y^4 (eu somei  e subtraí 2x^2y^2 mas não ajuda muito pois aparece 
depois sqrt2, na diferença de quadrados)
c)x^5+x^4+1

2)Equação:
x^2 + [x^2/(x+1)^2] =3

3)eq. em x.

srqt[a - sqrt(a+x)]=x

Bruno Leite

__
Get Your Private, Free Email at http://www.hotmail.com